EXAM 3: DAVIS

Réussis tes devoirs et examens dès maintenant avec Quizwiz!

A 72-year-old female client is brought to a clinic by her daughter because she has been experiencing fatigue, ataxia, and confusion. The laboratory report indicates that the client has a hemoglobin level of 7.0 g/dL. It is most important for the nurse to collect additional information about the client's: 1. food intake and medication use. 2. exposure to toxic chemicals. 3. ability to care for herself following treatment. 4. daily amount of alcohol consumed and last drink

1. food intake and medication use.

A nurse completes the measurements for the electrocardiogram (ECG) monitor rhythm strips for four different clients. Following analysis of the ECG rhythms, the nurse determines that one of the clients requires an immediate intervention. Which measurement findings warrant immediate intervention? 1. PRI = 0.20 sec.; QRS = 0.10 sec.; QT = 0.38 sec.; ventricular rate = 102 beats per minute (bpm) 2. P waves fibrillatory; PR unmeasureable; QRS = .08 sec.; QT unmeasureable; ventricular rate = 90 bpm 3. PRI inconsistent; more Ps than QRSs and no correlation between Ps and QRSs; QRS = .14 sec; atrial rate = 60 bpm; ventricular rate = 30 bpm 4. PRI = 0.24 sec.; QRS = .10 sec.; QT = .44 sec.; ventricular rate = 58 bpm

3. PRI inconsistent; more Ps than QRSs and no correlation between Ps and QRSs; QRS = .14 sec; atrial rate = 60 bpm; ventricular rate = 30 bpm

A child having recovered following ingestion of rat poison, develops aplastic anemia. Which assessment finding should a nurse conclude is specifically related to the aplastic anemia? SELECT ALL THAT APPLY. 1. Petechiae 2. Epistaxis 3. Easily fatigued 4. Pale skin color 5.Watery, itching eyes 6. Ulcerations around the mouth

ANSWER: 1, 2, 3, 4 Acquired aplastic anemia can result from chemicals, such as rat poisoning, that depress the hematopoietic activity in the bone marrow. Reduced platelet production results in easy bruising, petechiae, and bleeding, such as nose bleeds (epistaxis). The lower red blood cell count and resultant

A nurse is caring for a client with a spinal cord injury at the level of the sixth cervical vertebra. The client is at risk for the complication of autonomic dysreflexia. For which associated symptoms should a nurse monitor the client? SELECT ALL THAT APPLY. 1. Sweating 2. Headache 3. Hypotension 4. Blurred vision 5. Anxiety 6. Tachycardia

ANSWER: 1, 2, 4, 5 Autonomic dysreflexia results from uncontrolled sympathetic nervous system discharges causing vasodilation and hypertension. Sweating, headache, blurred vision, and anxiety occur secondary to the hypertension. Hypertension (not hypotension) is a symptom of autonomic dysreflexia. Bradycardia (not tachycardia) is secondary to vagal parasympathetic stimulation. ➧ Test-taking Tip: First, eliminate the cardiovascular symptoms listed in the options that are opposite symptoms associated with sympathetic nervous system stimulation. Then, select the options with the symptoms associated with severe hypertension. Finally, consider the symptoms associated with parasympathetic stimulation and select this option.

A nurse receives a serum laboratory report for six different clients with admitting diagnoses of chest pain. After reviewing all of the lab reports, in which order should the nurse address each lab value? Prioritize the order in which the nurse should address each of the clients' results. ______ Troponin T 42 ng/mL (0.0-0.4 ng/mL) ______ WBC 11,000 K/μL ______ Hgb 7.2 g/dL ______ SCr 2.2 mg/dL ______ K 2.2 mEq/L ______ Total cholesterol 430 mg/dL

ANSWER: 1, 6, 3, 4, 2, 5 The nurse should address the elevated troponin level first. Cardiospecific troponins (troponin T, cTnT, and troponin I, cTnI) are released into circulation after myocardial injury and are highly specific indicators of myocardial infarction. Since "time is muscle," the client needs to be treated immediately to prevent extension of the infarct and possible death. The nurse should address the decreased serum potassium level (K) second. The normal serum K level is 3.5 to 5.8 mEq/L. A low serum K level can cause life-threatening dysrhythmias. The normal hemoglobin (Hgb) is 13.1 to 17.1 g/dL. A low Hgb can contribute to inadequate tissue perfusion and contribute to myocardial ischemia. The normal serum creatinine (SCr) is 0.4 to 1.4 mg/dL. Impaired circulation may be causing this alteration and further client assessment is needed. Medication doses may need to be adjusted with impaired renal perfusion. The normal total serum cholesterol should be less than 200 mg/dL. This is a risk factor for development of coronary artery disease. The client needs teaching. The normal white blood cell (WBC) count is 3.9 to 11.9 K/μL. Because the finding is normal, it can be addressed last. ➧ Test-taking Tip: Knowledge of the normal ranges and the significance for critical laboratory values is expected on the NCLEX-RN®. Use the ABCs (airway, breathing, circulation) to determine priority. Of the laboratory values, determine those that are related to tissue perfusion (circulation),

A nurse is caring for a client hospitalized with idiopathic thrombocytopenic purpura (ITP). Which selfcare measures should the nurse plan to include when teaching the client? SELECT ALL THAT APPLY. 1. "Use dental floss after brushing your teeth to prevent gum hyperplasia." 2. "Use only an electric razor when shaving." 3. "Remove throw rugs in your home, and avoid clutter." 4. "Increase fiber in your diet, and drink plenty of liquids to avoid constipation." 5. "Keep appointments for monthly platelet transfusions."

ANSWER: 2, 3, 4

A nurse is teaching a client newly diagnosed with chronic stable angina. Which instructions should the nurse incorporate in the teaching session on measures to prevent future angina? SELECT ALL THAT APPLY. 1. Increase isometric arm exercises to build endurance. 2.Wear a face mask when outdoors in cold weather. 3. Take nitroglycerin before a stressful situation even though pain is not present. 4. Perform most exertional activities in the morning. 5. Avoid straining at stool. 6. Eliminate tobacco use.

ANSWER: 2, 3, 5, 6

41. Because a step-down cardiac unit is unusually busy, a nurse fails to obtain vital signs at 0200 hours for a client 2 days postoperative for a mitral valve replacement. The client was stable when assessed at 0600 hours, so the nurse documents the electrocardiogram monitor's heart rate in the client's medical record for both the 0400 and 0600 vital signs. The charge nurse supervising the nurse determines that the nurse's behavior was: SELECT ALL THAT APPLY. 1. the correct action because neither complications nor harmful effects occurred. 2. a legal issue because the nurse has fraudulently falsified documentation. 3. demonstrating beneficence because the nurse decided what was best for the client. 4. an ethical issue of veracity because the nurse has been untruthful regarding the client's care. 5. an ethical legal issue of confidentiality because the nurse disclosed incorrect information. 6. demonstrating distributive justice because the nurse decided other clients' needs were priority.

ANSWER: 2, 4 Documenting vital signs that the nurse did not obtain is both a legal and ethical concern because documents were falsified and the nurse was untruthful regarding obtaining the vital signs. Veracity is telling the truth and not lying or deceiving others. Even if harm had not occurred, the nurse's behavior of falsifying documentation poses an ethicallegal concern and is never the correct action. Beneficence is doing good. There is no information to indicate the nurse did what was best for the client. Confidentiality relates to privacy and not disclosing private information about another. Documenting incorrect vital signs is not disclosing confidential information. Distributive justice is the distribution of resources to clients. There is no information about the resources available to the nurse. ➧ Test-taking Tip: Focus on the nurse's behavior of falsifying documentation. Avoid reading into the question. Despite the unit being unusually busy, there is no information as to what the nurse was doing during the shift. Eliminate the options that are suggestive of nurse actions other than the behaviors presented.

After an inferior-septal wall myocardial infarction, which complication should a nurse suspect when noting jugular venous distention (JVD) and ascites? 1. Left-sided heart failure 2. Pulmonic valve malfunction 3. Right-sided heart failure 4. Ruptured septum

ANSWER: 3 Right-sided heart failure produces venous congestion in the systemic circulation resulting in JVD and ascites (from vascular congestion in the gastrointestinal tract). Additional signs include hepatomegaly, splenomegaly, and peripheral edema. Left-sided heart failure produces signs of pulmonary congestion, including crackles, S3 and S4 heart sounds, and pleural effusion. A characteristic finding of pulmonic valve malfunction would be a murmur. A murmur would also be auscultated with a ruptured septum, and the client would experience signs of cardiogenic shock. ➧ Test-taking Tip: Note that options 1 and 3 focus on different types of heart failure. Either one or both of these must be wrong. Focus on the client's signs of JVD and ascites and the cardiac anatomy to eliminate all but option 3.

After an inferior-septal wall myocardial infarction, which complication should a nurse suspect when noting jugular venous distention (JVD) and ascites? 1. Left-sided heart failure 2. Pulmonic valve malfunction 3. Right-sided heart failure 4. Ruptured septum 3.

ANSWER: 3 Right-sided heart failure produces venous congestion in the systemic circulation resulting in JVD and ascites (from vascular congestion in the gastrointestinal tract). Additional signs include hepatomegaly, splenomegaly, and peripheral edema. Left-sided heart failure produces signs of pulmonary congestion, including crackles, S3 and S4 heart sounds, and pleural effusion. A characteristic finding of pulmonic valve malfunction would be a murmur. A murmur would also be auscultated with a ruptured septum, and the client would experience signs of cardiogenic shock. ➧ Test-taking Tip: Note that options 1 and 3 focus on different types of heart failure. Either one or both of these must be wrong. Focus on the client's signs of JVD and ascites and the cardiac anatomy to eliminate all but option

A health-care provider documents, in the medical record of an 87-year-old hospitalized client, normal elder skin changes of senile purpura. The elder has no other skin changes. When assessing the client, which skin change, illustrated below, should the nurse expect to find?

Senile purpura is characterized by areas of ecchymosis and petechiae found on the hands, arms, and legs caused by the frail capillaries and decreased collagen support. It is a common skin lesion associated with aging. Seborrheic keratosis, option 1 (illustration A), is a benign pigmented lesion with a waxy surface on the face and trunk. Acrochordon, option 2 (illustration B), is a small, benign polyp-growth, also known as skin tags. Urticaria, option 4 (illustration D), is an abnormal lesion that can occur anywhere on the body from allergic reactions

Complications of MI-

are dysrhythmias

When a nurse removes a syringe after giving an intravenous (IV) push medication through a heparin lock port on a central line, the lock port adheres to the syringe and is removed from the central line catheter hub. The nurse notes that the clamp on the central line catheter is still open and suspects that air may have entered the central line catheter. Which immediate actions should be taken by the nurse? SELECT ALL THAT APPLY. 1. Notify the health-care provider 2. Close the clamp on the central line catheter 3. Turn the client onto the right side with the feet higher than the head 4. Attach a 10-mL syringe to the catheter, open the clamp, and aspirate until blood returns 5. Flush the line with 0.9% NaCl and then heparin if ordered

1. Notify the health-care provider 2. Close the clamp on the central line catheter 4. Attach a 10-mL syringe to the catheter, open the clamp, and aspirate until blood returns 5. Flush the line with 0.9% NaCl and then heparin if ordered

A nurse receives an order to administer a loop diuretic intravenous (IV) push to an adolescent client with hydronephrosis. For which adverse effects should the nurse monitor the client? SELECT ALL THAT APPLY. 1. Ototoxicity 2. Electrolyte abnormalities 3. Orthostatic hypotension 4. Hypertension 5. Hypoglycemia

1. Ototoxicity 2. Electrolyte abnormalities 3. Orthostatic hypotension

A client is on a heparin protocol preoperatively. Postoperatively a surgeon writes "Resume heparin at 1600, do not bolus." Then a cardiologist writes an order: "Restart heparin protocol no bolus previous rate at 0700 tomorrow if OK with surgeon." Later, an on-call surgeon writes "Clarification: Resume heparin drip tonight—no bolus at previous rate." At what rate should the heparin be started and why? Heparin Protocol 1. aPTT/PT; INR; daily CBC. 2. Initiate IV of 500 mL D5W with 25,000 units heparin at 25 mL/hr. 3. Give IV bolus of 2,500 units heparin (or ______units heparin). 4. Repeat aPTT/PT in 6 hours from time IV heparin infusion is initiated. 5. Adjust IV rate of heparin and repeat a PTT/PT per protocol 6. Notify physician if aPTT >129 sec.

1. aPTT/PT; INR; daily CBC.

Urine output=

1mL/kg/hr is the normal rate

A nurse is discharging a client following a myocardial infarction (MI) with stent placement and subsequent four-vessel coronary artery bypass graft surgery. The client has a body mass index (BMI) of 30, has a history of hypertension, smokes 1 pack per day (PPD) of cigarettes, and has prescriptions for aspirin, clopidogrel bisulfate (Plavix®), atenolol (Tenormin®), and atorvastatin (Lipitor®). Which discharge instructions are most appropriate? SELECT ALL THAT APPLY. 1. Discontinue the use of your elastic stockings (TEDS®) 2. Use a soft toothbrush and electric razor because you can bleed easily 3. Minimize your alcohol intake 4. Maintain your present weight to promote calorie intake for healing 5. Begin smoking cessation once your incision is completely healed 6. Discontinue the atenolol when your heart rate is less than 60 beats per minute

2. Use a soft toothbrush and electric razor because you can bleed easily 3. Minimize your alcohol intake

84. A nurse is discharging a client following a myocardial infarction (MI) with stent placement and subsequent four-vessel coronary artery bypass graft surgery. The client has a body mass index (BMI) of 30, has a history of hypertension, smokes 1 pack per day (PPD) of cigarettes, and has prescriptions for aspirin, clopidogrel bisulfate (Plavix®), atenolol (Tenormin®), and atorvastatin (Lipitor®). Which discharge instructions are most appropriate? SELECT ALL THAT APPLY. 1. Discontinue the use of your elastic stockings (TEDS®) 2. Use a soft toothbrush and electric razor because you can bleed easily 3. Minimize your alcohol intake 4. Maintain your present weight to promote calorie intake for healing 5. Begin smoking cessation once your incision is completely healed 6. Discontinue the atenolol when your heart rate is less than 60 beats per minute ans:

27. ANSWER: 2, 3 Both clopidogrel and aspirin prevent platelet aggregation and increase the risk for bleeding. Using bleeding precautions decreases the risk. Alcohol consumption increases caloric intake. A BMI of 30 indicates the client is overweight. Alcohol affects the liver and atorvastatin is metabolized by the liver and can increase liver enzymes. The elastic stockings help to decrease edema in the leg in which the saphenous vein was removed. Edema tends to increase with activity. A healthy diet for weight reduction can be initiated during the healing process. Interventions for smoking cessation should begin immediately, not after healing has occurred. Smoking causes a delay in healing. People who quit smoking after cardiac surgery reduce their risk of death by at least one-third. Atenolol should not be discontinued without a physician's order. Atenolol is used for treating hypertension and decreasing cardiac irritability after a MI and cardiac surgery. For a low heart rate, the client should consult with the physician and not discontinue the atenolol.

A client, with the diagnosis of congestive heart failure (CHF), contacts a clinic nurse and states, "I don't know why my ankles are so fat." What is the most appropriate initial response by the clinic nurse? 1. "Tell me about your activity level." 2. "What have you eaten in the last 24 hours?" 3. "Have you weighed yourself today?" 4. "How different are they from yesterday?"

3. "Have you weighed yourself today?"

A nurse is caring for a client who is to receive a unit of packed red blood cells (PRBCs). Which safety measure should the nurse implement when administering the blood transfusion? 1. Monitor vital signs every 15 minutes during the transfusion to detect complications early 2. Administer the unit of PRBCs slowly over 5 hours to prevent a transfusion reaction 3. Stop the transfusion if a reaction occurs and administer 0.9% NaCl at the IV catheter hub to keep the intravenous (IV) site patent 4. Deliver the PRBCs through an infusion pump with standard tubing to ensure a consistent rate

3. Stop the transfusion if a reaction occurs and administer 0.9% NaCl at the IV catheter hub to keep the intravenous (IV) site patent

Which action should a nurse take when caring for a client with thrombocytopenia? 1. Teach the client to use dental floss after teeth brushing to prevent dental caries 2. Treat an elevated temperature of 102°F (39°C) or greater with two 325-mg aspirin tablets 3. Prevent injury by maintaining the client on strict bedrest 4. Assess for signs of blurred vision, headache, or mental status changes

4. Assess for signs of blurred vision, headache, or mental status changes

A nurse is caring for a client who has been transferred from the coronary care unit following a myocardial infarction (MI). The client frequently calls the nurse to check on the ECG monitor rhythm. When the cardiac rehabilitation therapist arrives to transport the client for cardiac rehabilitation exercises, the client questions if he or she is ready for activity so soon after an MI. Which behavioral response should the nurse determine that the client is exhibiting? 1. Depression 2. Denial 3. Anger 4. Dependency

4. Dependency

97. A client, with the diagnosis of congestive heart failure (CHF), contacts a clinic nurse and states, "I don't know why my ankles are so fat." What is the most appropriate initial response by the clinic nurse? 1. "Tell me about your activity level." 2. "What have you eaten in the last 24 hours?" 3. "Have you weighed yourself today?" 4. "How different are they from yesterday?"

98. ANSWER: 3 Weight is a critical indicator of fluid loss or gain. Clients with CHF are taught to monitor weight daily at the same time using the same scale. Asking about activity level, foods eaten, and whether the client's ankles are different in appearance from the previous day are additional appropriate assessments and will direct necessary client education, but asking about the client's weight is the most appropriate question.

At 0730 hours, a nurse receives a verbal order for a cardiac catheterization to be completed on a client at 1400 hours. Which action should the nurse initiate first? 1. Initiate NPO (nothing per mouth) status for the client. 2. Teach the client about the procedure. 3. Start an intravenous (IV) infusion of 0.9% NaCl. 4. Ask the client to sign a consent form.

ANSWER: 1 A cardiac catheterization is an invasive procedure requiring the client to lie still in a supine position. The client is usually sedated with medication, such as midazolam (Versed®), during the procedure. To avoid aspiration, the client should be NPO 6 to 12 hours prior to the procedure. Because of the time element, NPO status should be initiated first and then teaching should occur. A consent form should be signed after the cardiologist has spoken with the client, and then an IV infusion order would be received. ➧ Test-taking Tip:The term "cardiac catheterization" in the stem indicates that this is an invasive procedure that has the potential to cause aspiration from sedation. Use the ABCs (airway, breathing, circulation) to determine which action should be first. Any action that pertains to maintaining a patent airway should be first.

At 0730 hours, a nurse receives a verbal order for a cardiac catheterization to be completed on a client at 1400 hours. Which action should the nurse initiate first? 1. Initiate NPO (nothing per mouth) status for the client. 2. Teach the client about the procedure. 3. Start an intravenous (IV) infusion of 0.9% NaCl. 4. Ask the client to sign a consent form.

ANSWER: 1 A cardiac catheterization is an invasive procedure requiring the client to lie still in a supine position. The client is usually sedated with medication, such as midazolam (Versed®), during the procedure. To avoid aspiration, the client should be NPO 6 to 12 hours prior to the procedure. Because of the time element, NPO status should be initiated first and then teaching should occur. A consent form should be signed after the cardiologist has spoken with the client, and then an IV infusion order would be received. ➧ Test-taking Tip:The term "cardiac catheterization" in the stem indicates that this is an invasive procedure that has the potential to cause aspiration from sedation. Use the ABCs (airway, breathing, circulation) to determine which action should be first. Any action that pertains to maintaining a patent airway should be first.

A client is admitted for coronary artery bypass surgery (CABG) with an anticipated admission to the coronary care unit (CCU). In preparation for the client's admission to the hospital, which action will best predict the sequence and timing of care and direct the course of the client's hospital stay? 1. Implementation of a clinical pathway 2. Initiation of a client education plan 3. Implementation of physician-initiated interventions 4. Initiation of discharge planning at the time of admission

ANSWER: 1 A clinical pathway is a standardized multidisciplinary care plan that projects the client goals, expected course of the client's treatment, and progress over the client's hospital stay. A client education plan will include the information required for the client to understand the care and treatment related to the surgical procedure but is not a comprehensive plan for their care. A physician-initiated intervention is an individual treatment initiated by a physician related to a medical diagnosis that is then carried out by a nurse. Although discharge planning helps establish client goals and should be started at the time of admission, it will not predict the client's course during the hospital stay. Test-taking Tip:The key words are "predict" and "direct." Apply knowledge of clinical pathways, care maps, clinical paths, collaborative care plans, or multidisciplinary care plans, all of which provide a timeline with projected client goals, expected course of the client's treatment, and progress over the client's hospital stay.

39. A nurse is caring for a client following a coronary artery bypass graft. Which assessment finding in the immediate postoperative period should be most concerning to the nurse? 1. No chest tube output for 1 hour when previously it was copious 2. Client temperature of 99.1°F (37.2°C) 3. Arterial blood gas (ABG) results show pH 7.32; Pco2 48; HCO3 28; Po2 80 4. Urine output of 160 mL in the last 4 hours

ANSWER: 1 A copiously draining chest tube that is no longer draining indicates an obstruction. There is an increased risk for cardiac tamponade or pleural effusion. A slight elevation in temperature could be the effects of rewarming after surgery. This should continue to be monitored, but is not immediately concerning. The ABG results show compensated respiratory acidosis. Though the pH is low and the PCO2 is high, the kidneys are compensating by conserving bicarbonate (HCO3). Normal pH is 7.35-7.45, PCO2 32-42 mm Hg, HCO3 20-24 mmol/L, and PO2 75-100 mm Hg. A urine output of 160 mL/4 hr is equivalent to 40 mL/hr; adequate, but it warrants continued monitoring. Less than 30 mL/hr indicates decreased renal function. ➧ Test-taking Tip:The key phrase in the question is "most concerning." Use the process of elimination and eliminate options 3 and 4 because these are normal findings. Of options 1 and 2, determine which option is most concerning.

A nurse is caring for a client following a coronary artery bypass graft. Which assessment finding in the immediate postoperative period should be most concerning to the nurse? 1. No chest tube output for 1 hour when previously it was copious 2. Client temperature of 99.1°F (37.2°C) 3. Arterial blood gas (ABG) results show pH 7.32; Pco2 48; HCO3 28; Po2 80 4. Urine output of 160 mL in the last 4 hours

ANSWER: 1 A copiously draining chest tube that is no longer draining indicates an obstruction. There is an increased risk for cardiac tamponade or pleural effusion. A slight elevation in temperature could be the effects of rewarming after surgery. This should continue to be monitored, but is not immediately concerning. The ABG results show compensated respiratory acidosis. Though the pH is low and the PCO2 is high, the kidneys are compensating by conserving bicarbonate (HCO3). Normal pH is 7.35-7.45, PCO2 32-42 mm Hg, HCO3 20-24 mmol/L, and PO2 75-100 mm Hg. A urine output of 160 mL/4 hr is equivalent to 40 mL/hr; adequate, but it warrants continued monitoring. Less than 30 mL/hr indicates decreased renal function. ➧ Test-taking Tip:The key phrase in the question is "most concerning." Use the process of elimination and eliminate options 3 and 4 because these are normal findings. Of options 1 and 2, determine which option is most concerning.

50. A homeless client, visiting a health clinic, is noted to have a smooth and reddened tongue and ulcers at the corners of the mouth. The client was tentatively diagnosed with a hematological disorder, and laboratory tests were prescribed. Based on this information, a nurse should expect the client's laboratory results to reveal: 1. low hemoglobin. 2. elevated red blood cells (RBCs). 3. prolonged prothrombin time (PT). 4. low white blood cells (WBCs).

ANSWER: 1 A smooth red tongue, ulcers at the corners of the mouth (angular cheilosis), and a low hemoglobin are signs of iron-deficiency anemia. Excess RBCs is associated with polycythemia vera. Prolonged PT is seen with clients taking anticoagulants or experiencing a coagulation disorder. Ulcers, if infected, would elevate the WBCs. ➧ Test-taking Tip: Think about the conditions that would produce the altered lab values in each option before answering the questions.

A homeless client, visiting a health clinic, is noted to have a smooth and reddened tongue and ulcers at the corners of the mouth. The client was tentatively diagnosed with a hematological disorder, and laboratory tests were prescribed. Based on this information, a nurse should expect the client's laboratory results to reveal: 1. low hemoglobin. 2. elevated red blood cells (RBCs). 3. prolonged prothrombin time (PT). 4. low white blood cells (WBCs).

ANSWER: 1 A smooth red tongue, ulcers at the corners of the mouth (angular cheilosis), and a low hemoglobin are signs of iron-deficiency anemia. Excess RBCs is associated with polycythemia vera. Prolonged PT is seen with clients taking anticoagulants or experiencing a coagulation disorder. Ulcers, if infected, would elevate the WBCs. ➧ Test-taking Tip: Think about the conditions that would produce the altered lab values in each option before answering the questions.

An 82-year-old client is hospitalized for the fifth time within a year. The client's problems include urinary track infection, myocardial infarction, stroke, and gastrointestinal bleeding. The client's physician recommends discharge to a nursing home, but the client refuses, stating, "As long as I can take care of myself, I'm not going into a home!" Assuming that the client is mentally competent, which ethical principle should be the primary guide for making decisions about supporting or rejecting the client's desire to return to home? 1. Autonomy 2. Beneficence 3. Nonmalfeasance 4. Justice

ANSWER: 1 Autonomy is one's personal freedom to direct his or her own life as long as it does not infringe on the rights of others, thus letting the client decide. The principle of beneficence means attempting to do good for others, while nonmalfeasance is the principle of avoiding or preventing harm. Justice is a term meaning "giving to each what is their due" and, in health care, is the principle of fair distribution of health care in accordance to need and benefits it can provide.

38. A client admitted with unstable angina is started on intravenous heparin and nitroglycerin. The client's chest pain resolves, and the client is weaned from the nitroglycerin. Noting that the client had a synthetic valve replacement for aortic stenosis 2 years ago, a physician writes an order to restart the oral warfarin (Coumadin®) 5 mg at 1900 hours. Which is the nurse's best action? 1. Administer the warfarin as prescribed. 2. Call the physician to question the warfarin order. 3. Discontinue the heparin drip and then administer the warfarin. 4. Hold the dose of warfarin until the heparin has been discontinued.

ANSWER: 1 Both heparin and warfarin are anticoagulants, but their actions are different. Oral warfarin requires 3 to 5 days to reach effective levels. It is usually begun while the client is still on heparin. Calling the physician is unnecessary. The nurse's scope of practice does not permit altering medication orders. The nurse should neither discontinue the heparin nor hold the warfarin without a written order. ➧ Test-taking Tip: Use the process of elimination to eliminate options 3 and 4, which alter medication orders, because these are not within the nurse's scope of practice. Of the two remaining options, focus on the action of heparin and warfarin. Recall that warfarin takes 3 to 5 days to reach therapeutic effectiveness, during which time the client will continue to require anticoagulation.

A client admitted with unstable angina is started on intravenous heparin and nitroglycerin. The client's chest pain resolves, and the client is weaned from the nitroglycerin. Noting that the client had a synthetic valve replacement for aortic stenosis 2 years ago, a physician writes an order to restart the oral warfarin (Coumadin®) 5 mg at 1900 hours. Which is the nurse's best action? 1. Administer the warfarin as prescribed. 2. Call the physician to question the warfarin order. 3. Discontinue the heparin drip and then administer the warfarin. 4. Hold the dose of warfarin until the heparin has been discontinued.

ANSWER: 1 Both heparin and warfarin are anticoagulants, but their actions are different. Oral warfarin requires 3 to 5 days to reach effective levels. It is usually begun while the client is still on heparin. Calling the physician is unnecessary. The nurse's scope of practice does not permit altering medication orders. The nurse should neither discontinue the heparin nor hold the warfarin without a written order. ➧ Test-taking Tip: Use the process of elimination to eliminate options 3 and 4, which alter medication orders, because these are not within the nurse's scope of practice. Of the two remaining options, focus on the action of heparin and warfarin. Recall that warfarin takes 3 to 5 days to reach therapeutic effectiveness, during which time the client will continue to require anticoagulation.

77. An adult client taking warfarin (Coumadin®) for treatment of atrial fibrillation presents to an emergency department complaining of weakness and fatigue. The client's skin is pale and diaphoretic, and the blood pressure is 85/58 mm Hg. When the client is attached to a cardiac monitor, the client's rhythm is atrial flutter with a ventricular rate of 140 beats per minute. A nurse should expect to initiate health-care provider's orders to treat: 1. noncoronary cardiogenic shock. 2. coronary cardiogenic shock. 3. hypovolemic shock. 4. neurogenic shock.

ANSWER: 1 Cardiogenic shock occurs as a result of inadequate supply of oxygen to the heart and tissues due to the heart's inability to contract and pump blood. Noncoronary cardiogenic shock results from conditions that stress the myocardium or result in ineffective myocardial function, such as dysrhythmias. Coronary cardiogenic shock occurs when there is significant damage to the left ventricle such as from an anterior wall myocardial infarction. Though the client is taking warfarin, which can cause bleeding and hypovolemic shock from a decrease in intravascular volume, bleeding is not reported. In neurogenic shock, vasodilatation occurs from a loss of balance between parasympathetic and sympathetic stimulation. ➧ Test-taking Tip: Both the client's symptoms and the options suggest shock. Because the cardiac monitor shows a change in rhythm and there is no evidence of bleeding, the shock is likely cardiac in origin. Eliminate options 3 and 4. Use the word "coronary" to eliminate one of the remaining two options. The coronary vessels supply blood to the heart. Because atrial flutter does not involve vessels, eliminate option 2.

An adult client taking warfarin (Coumadin®) for treatment of atrial fibrillation presents to an emergency department complaining of weakness and fatigue. The client's skin is pale and diaphoretic, and the blood pressure is 85/58 mm Hg. When the client is attached to a cardiac monitor, the client's rhythm is atrial flutter with a ventricular rate of 140 beats per minute. A nurse should expect to initiate health-care provider's orders to treat: 1. noncoronary cardiogenic shock. 2. coronary cardiogenic shock. 3. hypovolemic shock. 4. neurogenic shock.

ANSWER: 1 Cardiogenic shock occurs as a result of inadequate supply of oxygen to the heart and tissues due to the heart's inability to contract and pump blood. Noncoronary cardiogenic shock results from conditions that stress the myocardium or result in ineffective myocardial function, such as dysrhythmias. Coronary cardiogenic shock occurs when there is significant damage to the left ventricle such as from an anterior wall myocardial infarction. Though the client is taking warfarin, which can cause bleeding and hypovolemic shock from a decrease in intravascular volume, bleeding is not reported. In neurogenic shock, vasodilatation occurs from a loss of balance between parasympathetic and sympathetic stimulation. ➧ Test-taking Tip: Both the client's symptoms and the options suggest shock. Because the cardiac monitor shows a change in rhythm and there is no evidence of bleeding, the shock is likely cardiac in origin. Eliminate options 3 and 4. Use the word "coronary" to eliminate one of the remaining two options. The coronary vessels supply blood to the heart. Because atrial flutter does not involve vessels, eliminate option 2.

Which low-potassium foods (less than 400 mg of potassium per serving) should a nurse plan to include on a list of acceptable foods for a client experiencing chronic renal failure? 1. Cranberry juice, grapes, fresh string beans, fortified puffed rice cereal 2. Prune juice, dried fruit, tomatoes, and all-bran cereal 3. Milk, cantaloupe, peas, and granola cereal 4. Orange juice, raisins, spinach, and dried beans

ANSWER: 1 Cranberry juice, grapes, fresh string beans, and fortified puffed rice cereal are low-potassium foods. Foods that should be restricted include cantaloupe, tomatoes, prune juice, milk, dried fruit, bananas, and dried beans. Other high-potassium foods include avocados, brussels sprouts, peas, raisins, spinach, winter squash, molasses, all-bran cereal, and nuts. Salt substitutes that contain potassium chloride should also be avoided. ➧ Test-taking Tip: Identify key high-potassium foods in each option (tomatoes, cantaloupe, and orange juice), and use the process of elimination.

86. A 72-year-old female client is brought to a clinic by her daughter because she has been experiencing fatigue, ataxia, and confusion. The laboratory report indicates that the client has a hemoglobin level of 7.0 g/dL. It is most important for the nurse to collect additional information about the client's: 1. food intake and medication use. 2. exposure to toxic chemicals. 3. ability to care for herself following treatment. 4. daily amount of alcohol consumed and last drink. 36.

ANSWER: 1 Fatigue, ataxia, confusion, and a hemoglobin level of 7.0 g/dL suggests an anemia. The majority of anemias in older adults are due to nutritional deficiencies and medication use for chronic conditions. Exposure to chemical toxins is less common than nutritional deficiencies as a cause of anemia in older adults. The symptoms of ataxia and confusion can occur because of inadequate tissue oxygenation from the low hemoglobin. The symptoms are likely to resolve with blood replacement. Although alcohol consumption can cause ataxia and confusion and falls could cause bleeding, the situation does not indicate that the client has bruising or has been bleeding.

A 72-year-old client with a deep vein thrombosis in the left leg and a history of a brain tumor is hospitalized for 3 days. The client's care plan indicates a nursing diagnosis of Imbalanced nutrition: less than body requirements related to poor appetite and decreased oral intake. Which assessment finding would best indicate a need to revise the care plan related to the nursing diagnosis? 1. Oral mucous membranes are dry due to dehydration. 2. Daily intake and output reveals that daily caloric intake is inadequate. 3. Client is not receptive to education regarding nutrition. 4. Client states that he/she is not hungry.

ANSWER: 1 If the client has an impaired oral mucous membrane, it may be uncomfortable to chew and swallow food, therefore exacerbating the underlying condition. To help the client achieve the goal of balanced nutrition, addressing the problem on the care plan will alert caregivers to the issue and facilitate appropriate interventions. Although the other three answers may also support the client's nutrition needs, revising the care plan to address the impaired oral mucous membrane is the best action to take related to the diagnosis. ➧ Test-taking Tip:The key words are "best indicate." Select an option that would exacerbate the nursing diagnosis.

nurse should assess a client with hemolytic anemia for weakness, fatigue, malaise, skin and mucous membrane pallor, and: 1. jaundice. 2. a smooth red tongue. 3. a craving for ice. 4. a poor intake of fresh vegetables.

ANSWER: 1 Jaundice occurs from the shortened life span of the red blood cell and the breakdown of hemoglobin. About 80% of heme is converted to bilirubin, conjugated in the liver, and excreted in the bile. The increased bilirubin in the blood causes the jaundice. A smooth, red tongue and a craving for ice are seen with iron-deficiency anemia. Folate deficiency occurs in people who rarely eat fresh vegetables. ➧ Test-taking Tip: Use knowledge of medical terminology, recalling that "-lytic" is destruction of red blood cells

52. A nurse should assess a client with hemolytic anemia for weakness, fatigue, malaise, skin and mucous membrane pallor, and: 1. jaundice. 2. a smooth red tongue. 3. a craving for ice. 4. a poor intake of fresh vegetables.

ANSWER: 1 Jaundice occurs from the shortened life span of the red blood cell and the breakdown of hemoglobin. About 80% of heme is converted to bilirubin, conjugated in the liver, and excreted in the bile. The increased bilirubin in the blood causes the jaundice. A smooth, red tongue and a craving for ice are seen with iron-deficiency anemia. Folate deficiency occurs in people who rarely eat fresh vegetables. ➧ Test-taking Tip: Use knowledge of medical terminology, recalling that "-lytic" is destruction of red blood cells.

66. A nurse who is initiating an intravenous infusion of lactated Ringer's (LR) for a client in shock recognizes that the purpose of LR for the client is to: 1. increase fluid volume and urinary output. 2. draw water from the cells into the blood vessels. 3. provide dextrose and nutrients to prevent cellular death. 4. replace electrolytes of sodium, potassium, calcium, and magnesium for cardiac stabilization.

ANSWER: 1 LR is an isotonic crystalloid solution containing multiple electrolytes in approximately the same concentration as plasma. It enters the cells from the blood, replacing fluids and increasing urinary output. A hypertonic solution draws fluid from the cells into the vascular compartment. LR alone does not contain dextrose. Formulations with dextrose are available. Magnesium is not a component of LR. ➧ Test-taking Tip: Apply knowledge of the components of LR and use the process of elimination. Eliminate options 3 and 4 because neither dextrose nor magnesium is a component of LR.

A nurse who is initiating an intravenous infusion of lactated Ringer's (LR) for a client in shock recognizes that the purpose of LR for the client is to: 1. increase fluid volume and urinary output. 2. draw water from the cells into the blood vessels. 3. provide dextrose and nutrients to prevent cellular death. 4. replace electrolytes of sodium, potassium, calcium, and magnesium for cardiac stabilization.

ANSWER: 1 LR is an isotonic crystalloid solution containing multiple electrolytes in approximately the same concentration as plasma. It enters the cells from the blood, replacing fluids and increasing urinary output. A hypertonic solution draws fluid from the cells into the vascular compartment. LR alone does not contain dextrose. Formulations with dextrose are available. Magnesium is not a component of LR. ➧ Test-taking Tip: Apply knowledge of the components of LR and use the process of elimination. Eliminate options 3 and 4 because neither dextrose nor magnesium is a component of LR.

A nurse receives the following medication orders while caring for multiple clients. Which medication should the nurse plan to administer first? 1. Nitroglycerin (Nitrostat®) 0.4 mg sublingually (SL) stat for the client experiencing chest pain 2. Morphine sulfate 4 mg intravenously (IV) now for the client experiencing incisional pain 3. Lorazepam 2 mg IV now for the client experiencing restlessness and picking at tubing 4. One unit packed red blood cells stat for the client with a hemoglobin of 9.5 g

ANSWER: 1 Nitroglycerin increases coronary blood flow by dilating coronary arteries and improving collateral flow to ischemic areas of the heart. Increasing collateral blood flow reduces anginal pain and the potential of myocardial infarction. This action has the greatest potential of changing client outcomes and can be performed more quickly than the other actions. Both morphine and lorazepam are controlled substances, requiring the nurse to retrieve and sign these out from a secure location. Administering IV medications takes longer than SL medications. Obtaining blood from the blood bank will take longer than the time it takes to administer a SL medication.

A nurse receives the following medication orders while caring for multiple clients. Which medication should the nurse plan to administer first? 1. Nitroglycerin (Nitrostat®) 0.4 mg sublingually (SL) stat for the client experiencing chest pain 2. Morphine sulfate 4 mg intravenously (IV) now for the client experiencing incisional pain 3. Lorazepam 2 mg IV now for the client experiencing restlessness and picking at tubing 4. One unit packed red blood cells stat for the client with a hemoglobin of 9.5 g

ANSWER: 1 Nitroglycerin increases coronary blood flow by dilating coronary arteries and improving collateral flow to ischemic areas of the heart. Increasing collateral blood flow reduces anginal pain and the potential of myocardial infarction. This action has the greatest potential of changing client outcomes and can be performed more quickly than the other actions. Both morphine and lorazepam are controlled substances, requiring the nurse to retrieve and sign these out from a secure location. Administering IV medications takes longer than SL medications. Obtaining blood from the blood bank will take longer than the time it takes to administer a SL medication. ➧ Test-taking Tip: Use the ABCs (airway, breathing, circulation) and the time it takes to implement each action to establish the priority. Because nitroglycerine affects the circulatory system,

A nurse is caring for an older adult client who is asking for nutritional information. Good nutrition advice for the older adult should be to: 1. maintain an appropriate weight for height and include nutrient-dense foods. 2. increase vitamin E intake and exercise daily. 3. avoid high-fiber foods and take a multiplevitamin supplement. 4. become a vegetarian and drink 2 quarts of water per day.

ANSWER: 1 Overall weight control and consumption of foods high in nutrients will promote healthy aging. Supplements, such as vitamin E, are not substitutes for food. Fiber is only part of a healthy diet. A vegan diet does not ensure a nutrient-dense diet. ➧ Test-taking Tip: Consider the broad picture of healthy aging before selecting an option.

A nurse receives an order to administer digoxin to a 6-year-old child. Prior to administering the medication, the nurse reviews the child's laboratory report. Which laboratory value should concern the nurse and be reported to the physician? 1. K 3.2 mEq/L 2. Hgb 10 g/dL 3. Digoxin level 1.8 ng/mL 4. Creatinine 0.3 mg/dL

ANSWER: 1 The low serum potassium level should concern the nurse and be reported to the physician. A low serum potassium level would increase the risk of digoxin toxicity. Although the hemoglobin level is a little low, this is not most concerning. The digoxin level is on the high side of normal, thus administering digoxin while the serum potassium level is low increases the risk further. The serum creatinine level is a measure of renal function. It is within normal limits and not concerning

A nurse is caring for a pediatric client immediately following a permanent pacemaker placement. Which intervention should be the nurse's first priority for this client? 1. Initiate continuous electrocardiogram (ECG) monitoring. 2. Administer only non-narcotic analgesic medications to avoiding masking signs and symptoms of complications. 3. Transport the child to the radiology department for a chest x-ray. 4. Administer antibiotic therapy to prevent infection.

ANSWER: 1 The nurse's first priority should be to initiate continuous ECG monitoring. Continuous ECG monitoring during the recovery phase is important to assess pacemaker function immediately following placement. Analgesics, including narcotics, are administered as needed to control pain. A chest x-ray is performed within 24 hours for future comparison, but it is not the priority. While the nurse should carefully monitor the site for signs of infection; prophylactic antibiotic therapy is not the priority. ➧ Test-taking Tip: Use the ABCs (airway, breathing, circulation) to establish priority. Recall that the pacemaker initiates an impulse to stimulate the electrical conduction of a person's heart

68. A nurse is caring for a pediatric client who has congestive heart failure (CHF). The client is receiving digoxin therapy. Which laboratory test result is most important to evaluate when preparing to administer digoxin? 1. Serum potassium levels 2. Serum magnesium levels 3. Serum sodium levels 4. Serum chloride levels

ANSWER: 1 The serum potassium level is the most important result when preparing to administer digoxin. Hypokalemia increases the risk of digoxin toxicity and life-threatening dysrhythmias. Though important, magnesium, sodium, and chloride levels are not as essential. ➧ Test-taking Tip: Think of the electrolyte that

A nurse is caring for a pediatric client who has congestive heart failure (CHF). The client is receiving digoxin therapy. Which laboratory test result is most important to evaluate when preparing to administer digoxin? 1. Serum potassium levels 2. Serum magnesium levels 3. Serum sodium levels 4. Serum chloride levels

ANSWER: 1 The serum potassium level is the most important result when preparing to administer digoxin. Hypokalemia increases the risk of digoxin toxicity and life-threatening dysrhythmias. Though important, magnesium, sodium, and chloride levels are not as essential. ➧ Test-taking Tip: Think of the electrolyte that would have the greatest effect on the heart. Applying knowledge of basic pharmacological principles will direct you to the correct response.

96. A hospice nurse is caring for a 4-year-old client diagnosed with a terminal, congenital heart defect who is dying. In meeting the psychosocial needs of the client and family, the nurse employs a variety of techniques. Which nursing action demonstrates uncaring behaviors? 1. Minimally touching the client and conversing, in a soft voice, as if the client is alert 2. Allowing the client and family privacy to express feelings and comfort one another 3. Encouraging the family to talk with and reassure the child who is dying 4. Encouraging the client and family to verbalize feelings of sadness, loss, and forgiveness

ANSWER: 1 When a client is near death, they may seem to be withdrawn from the physical environment, maintaining the ability to hear while not able to respond. The nurse should touch the client gently (not minimally) and converse as if the client is alert, using a soft voice. Allowing for privacy and encouraging talking, reassuring, and verbalizing feelings are important elements of saying goodbye.

59. A client with symptoms of anemia and a hemoglobin of 7.8 g/dL refuses blood and blood products transfusions for religious reasons. A nurse should anticipate that a health-care provider might prescribe: SELECT ALL THAT APPLY. 1. Epoetin alfa (Procrit®) 2. Folic acid 3. Albumin 4. Platelets 5. Fresh frozen plasma 6. Granulocytes

ANSWER: 1, 2 Epoetin alfa (erythropoietin growth factor) and folic acid promote erythropoiesis (production of red blood cells), thus decreasing the need for transfusions. Folic acid also stimulates production of white blood cells and platelets. According to the evidence base, for persons with hemoglobin less than 8 g/dL, the use of either transfusion or erythropoietic growth factor was rated "appropriate." Albumin, platelets, plasma, and granulocytes are all blood products.

A client with symptoms of anemia and a hemoglobin of 7.8 g/dL refuses blood and blood products transfusions for religious reasons. A nurse should anticipate that a health-care provider might prescribe: SELECT ALL THAT APPLY. 1. Epoetin alfa (Procrit®) 2. Folic acid 3. Albumin 4. Platelets 5. Fresh frozen plasma 6. Granulocytes

ANSWER: 1, 2 Epoetin alfa (erythropoietin growth factor) and folic acid promote erythropoiesis (production of red blood cells), thus decreasing the need for transfusions. Folic acid also stimulates production of white blood cells and platelets. According to the evidence base, for persons with hemoglobin less than 8 g/dL, the use of either transfusion or erythropoietic growth factor was rated "appropriate." Albumin, platelets, plasma, and granulocytes are all blood products. ➧ Test-taking Tip: Focus on the issue: low hemoglobin and client refuses blood and blood products. Select options 1 and 2, because these are not blood products.

A client is hospitalized with a diagnosis of sickle cell crisis. Which findings should lead a nurse to conclude that outcomes have been achieved for this client? SELECT ALL THAT APPLY. 1. Leukocyte count 7,500/mm3 2. Describes the importance of keeping warm 3. Acute pain controlled at less than 3 on a 0 to 10 scale with analgesics 4. Free of chest pain or dyspnea 5. Blood transfusions effective in diminishing cell sickling 6. Hydroxyurea (Hydrea®) effective in suppressing leukocyte formation

ANSWER: 1, 2, 3, 4 A leukocyte count of 7,500/mm3 is within normal range (5,000 to 10,000/mm3 indicates the absence of an infection). Keeping warm and avoiding chills will help to prevent infection. Also, cold causes vasoconstriction, slowing blood flow and aggravating the sickling process. Acute pain is due to tissue hypoxia from the agglutination of sickled cells within blood vessels. Acute chest syndrome and pulmonary hypertension are two of the many complications associated with sickle cell disease. Red blood cell transfusions may help to prevent complications, but transfusions do not alter the person's body from producing the deformed erythrocytes. Hydroxyurea can decrease the permanent formation of sickled cells. A side effect (not therapeutic effect) of Hydroxyurea is suppression of leukocyte formation. ➧ Test-taking Tip: Remember that outcomes need to be measurable. Options 5 and 6 are not measurable.

82. A nurse is caring for a 2-year-old child at risk for iron-deficiency anemia. To prevent iron-deficiency anemia in toddlers, a nurse should recommend: SELECT ALL THAT APPLY. 1. limiting the toddler's milk intake to 24 ounces per day. 2. limiting the toddler's juice intake to 4 to 6 ounces per day. 3. offering iron-rich foods such as beef, lentils, broccoli, and raisins. 4. iron supplementation for 1- to 2-year-old children of parents in a lower economic class. 5. avoiding vegan diets for all toddlers. 6. parental feeding of the toddler to ensure an adequate intake.

ANSWER: 1, 2, 3, 4 To maintain the appetite for iron-enriched cereals, meats, and iron-rich fruits and vegetables, milk should be limited to 24 ounces per day and juice to 4 to 6 ounces per day for children aged 1 to 5 years. Beef, lentils, broccoli, and raisins are some of the iron-rich foods. In a pilot study involving poor and minority populations, prophylactic daily iron supplementation was estimated to reduce the incidence of iron-deficiency anemia by 72%.

A nurse assesses the coping-stress tolerance in the functional health assessment of a client diagnosed with anemia. The nurse notes that the client has a copingstress tolerance problem. Which nursing diagnoses, pertaining to the client's coping-stress tolerance pattern, should the nurse document in the client's plan of care? SELECT ALL THAT APPLY. 1. Caregiver role strain 2. Defensive coping 3. Relocation stress syndrome 4. Stress overload 5. Ineffective coping 6. Readiness for enhanced coping

ANSWER: 1, 2, 3, 4, 5 Caregiver role strain, defensive coping, relocation stress syndrome, stress overload, and ineffective coping are all applicable to the copingstress tolerance pattern. Each has different defining characteristics and variable interventions. Thus, the nurse should collect additional data. Readiness for enhanced coping indicates that the client is showing signs of resolution of a problem.

A new nurse is managing the care of a pediatric client preparing for a cardiac catheterization under the supervision of an experienced nurse. Which factor identified by the new nurse demonstrates an understanding of the information that can be collected during cardiac catheterization? SELECT ALL THAT APPLY 1. Oxygen saturation of blood within the chambers and great vessels 2. Pressure of blood flow within the heart chambers 3. Cardiac output (CO) 4. Anatomic abnormalities 5. Ankle brachial index (ABI) 6. Ejection fraction .

ANSWER: 1, 2, 3, 4, 6 In cardiac catheterization, a small radiopaque catheter is passed through the major vein in the arm, leg, or neck into the heart. Blood specimens can be obtained to determine oxygen saturation levels, and contrast dye can be injected for angiography and to assess for anatomic abnormalities such as septal defects or obstruction of flow. Pressure of blood flow in the heart chambers, CO, stroke volume, and ejection fraction can be evaluated during the procedure. ABI is a ratio of the ankle systolic pressure to the arm systolic pressure and an objective measurement of arterial disease that quantifies the degree of stenosis. It is not related to a cardiac catheterization procedure. ➧ Test-taking Tip: Apply knowledge of a cardiac catheterization procedure to answer this question. Eliminate the one option that is unrelated to a cardiac catheterization

A new nurse is managing the care of a pediatric client preparing for a cardiac catheterization under the supervision of an experienced nurse. Which factor identified by the new nurse demonstrates an understanding of the information that can be collected during cardiac catheterization? SELECT ALL THAT APPLY 1. Oxygen saturation of blood within the chambers and great vessels 2. Pressure of blood flow within the heart chambers 3. Cardiac output (CO) 4. Anatomic abnormalities 5. Ankle brachial index (ABI) 6. Ejection fraction

ANSWER: 1, 2, 3, 4, 6 In cardiac catheterization, a small radiopaque catheter is passed through the major vein in the arm, leg, or neck into the heart. Blood specimens can be obtained to determine oxygen saturation levels, and contrast dye can be injected for angiography and to assess for anatomic abnormalities such as septal defects or obstruction of flow. Pressure of blood flow in the heart chambers, CO, stroke volume, and ejection fraction can be evaluated during the procedure. ABI is a ratio of the ankle systolic pressure to the arm systolic pressure and an objective measurement of arterial disease that quantifies the degree of stenosis. It is not related to a cardiac catheterization procedure. ➧ Test-taking Tip: Apply knowledge of a cardiac catheterization procedure to answer this question. Eliminate the one option that is unrelated to a cardiac catheterization.

A nurse is caring for multiple 25-year-old female clients. For which clients should the nurse plan to obtain a referral for genetic counseling and family planning? SELECT ALL THAT APPLY. 1. Client diagnosed with thalassemia major 2. Client diagnosed with sickle cell disease 3. Client diagnosed with hemophilia A 4. Client diagnosed with autoimmune hemolytic anemia 5. Client diagnosed with hemophilia B

ANSWER: 1, 2, 3, 5 Thalassemia, sickle cell disease, and hemophilia A and B are hereditary disorders. Autoimmune hemolytic anemia is an acquired hemolytic anemia. ➧ Test-taking Tip:The key term is "genetic counseling." Eliminate an acquired condition.

58. A nurse working in the blood mobile is screening clients to determine if they qualify for blood donation of whole blood. Which questions should the nurse ask during the screening interview? SELECT ALL THAT APPLY. 1. "What is your age?" 2. "If you have a tattoo, when did you receive it?" 3. "Have you had any close contact with anyone with HIV or hepatitis?" 4. "If you smoke, when was the last time you smoked tobacco products?" 5. "Have you been immunized for rubella, mumps, or varicella within the last month?" 6. "Did you receive a blood transfusion or blood product anywhere outside of the United States?"

ANSWER: 1, 2, 3, 5, 6 Persons ineligible to donate blood include those younger than 17 years of age; those immunized for rubella, mumps, or varicella within the last month; those with a history of a recent tattoo or close contact with a person with HIV or hepatitis; and those receiving transfusions in the United Kingdom, Gibraltar, or the Falkland Islands because of the increased likelihood of transmitting Creutzfeldt-Jakob disease. Persons who smoke tobacco products may donate blood unless they have a recent history of asthma.

nurse working in the blood mobile is screening clients to determine if they qualify for blood donation of whole blood. Which questions should the nurse ask during the screening interview? SELECT ALL THAT APPLY. 1. "What is your age?" 2. "If you have a tattoo, when did you receive it?" 3. "Have you had any close contact with anyone with HIV or hepatitis?" 4. "If you smoke, when was the last time you smoked tobacco products?" 5. "Have you been immunized for rubella, mumps, or varicella within the last month?" 6. "Did you receive a blood transfusion or blood product anywhere outside of the United States?"

ANSWER: 1, 2, 3, 5, 6 Persons ineligible to donate blood include those younger than 17 years of age; those immunized for rubella, mumps, or varicella within the last month; those with a history of a recent tattoo or close contact with a person with HIV or hepatitis; and those receiving transfusions in the United Kingdom, Gibraltar, or the Falkland Islands because of the increased likelihood of transmitting Creutzfeldt-Jakob disease. Persons who smoke tobacco products may donate blood unless they have a recent history of asthma. ➧ Test-taking Tip: Focus on infectious disease transmission, noting that options 2, 3, 5, and 6 increase the risk of transmitting the diseases.

A nurse should anticipate instructing a client scheduled for a coronary artery bypass graft to: SELECT ALL THAT APPLY. 1. discontinue taking aspirin prior to surgery. 2. perform postoperative cardiac rehabilitation exercises and stress management strategies. 3. wash with an antimicrobial soap the evening prior to surgery. 4. shave the chest and legs and then shower to remove the hair. 5. resume normal activities when discharged from the hospital. 6. expect close monitoring after surgery, several intravenous (IV) lines, a urinary catheter, endotracheal tube, and chest tubes. y.

ANSWER: 1, 2, 3, 6 Aspirin decreases platelet aggregation and increases the risk of bleeding. It is usually discontinued a few days prior to surgery. A postoperative cardiac rehabilitation program is begun usually on the second postoperative day and includes exercises and stress management. The client should use an antimicrobial soap when showering or bathing the evening before and the day of surgery to decrease the risk of infection. Teaching about expectations of close monitoring, IV lines, a urinary catheter, endotracheal tube, and chest tubes can reduce client and family anxiety. The client may be offered a tour of the critical care unit prior to surgery or be given videos to view. Although the client's skin will be shaved, this will be completed just prior to surgery to avoid nicks and decrease the risk of infection. Activities that stress the sternum, such as lifting, driving, and overhead reaching, will be restricted after surgery. ➧ Test-taking Tip: Use the process of elimination to eliminate options 4 and 5 because these increase surgical risk and the risk of complications after surgerY

40. A nurse should anticipate instructing a client scheduled for a coronary artery bypass graft to: SELECT ALL THAT APPLY. 1. discontinue taking aspirin prior to surgery. 2. perform postoperative cardiac rehabilitation exercises and stress management strategies. 3. wash with an antimicrobial soap the evening prior to surgery. 4. shave the chest and legs and then shower to remove the hair. 5. resume normal activities when discharged from the hospital. 6. expect close monitoring after surgery, several intravenous (IV) lines, a urinary catheter, endotracheal tube, and chest tubes.

ANSWER: 1, 2, 3, 6 Aspirin decreases platelet aggregation and increases the risk of bleeding. It is usually discontinued a few days prior to surgery. A postoperative cardiac rehabilitation program is begun usually on the second postoperative day and includes exercises and stress management. The client should use an antimicrobial soap when showering or bathing the evening before and the day of surgery to decrease the risk of infection. Teaching about expectations of close monitoring, IV lines, a urinary catheter, endotracheal tube, and chest tubes can reduce client and family anxiety. The client may be offered a tour of the critical care unit prior to surgery or be given videos to view. Although the client's skin will be shaved, this will be completed just prior to surgery to avoid nicks and decrease the risk of infection. Activities that stress the sternum, such as lifting, driving, and overhead reaching, will be restricted after surgery. ➧ Test-taking Tip: Use the process of elimination to eliminate options 4 and 5 because these increase surgical risk and the risk of complications after surgery.

72. A nurse is managing the care of a pediatric client in congestive heart failure (CHF). Which medically delegated interventions should be included in the care of the client? SELECT ALL THAT APPLY. 1. Oral positive inotropic agents 2. Diuretics 3. ACE inhibitors 4. Hypolipidemic agents 5. Oral positive chronotropic agents 6. Beta blockers

ANSWER: 1, 2, 3, 6 Therapeutic management of a client with congestive heart failure may include oral positive inotropic agents, diuretics, ACE inhibitors, and beta blockers. Positive inotropic agents increase the strength of muscular contraction. Diuretics increase urine excretion and reduce volume overload. ACE inhibitors (antihypertensive agents) relax arteries and promote renal excretion of salt and water by inhibiting the activity of an angiotensin-converting enzyme. Beta blockers decrease the workload of the heart by decreasing the amount of pressure against which it has to pump. Hypolipidemic agents are not standard management of CHF for pediatric clients. These agents are used to treat hyperlipidemia (elevated cholesterol and triglycerides), which contribute to coronary artery disease. Positive chronotropic agents will increase the heart rate.

A nurse is managing the care of a pediatric client in congestive heart failure (CHF). Which medically delegated interventions should be included in the care of the client? SELECT ALL THAT APPLY. 1. Oral positive inotropic agents 2. Diuretics 3. ACE inhibitors 4. Hypolipidemic agents 5. Oral positive chronotropic agents 6. Beta blockers

ANSWER: 1, 2, 3, 6 Therapeutic management of a client with congestive heart failure may include oral positive inotropic agents, diuretics, ACE inhibitors, and beta blockers. Positive inotropic agents increase the strength of muscular contraction. Diuretics increase urine excretion and reduce volume overload. ACE inhibitors (antihypertensive agents) relax arteries and promote renal excretion of salt and water by inhibiting the activity of an angiotensin-converting enzyme. Beta blockers decrease the workload of the heart by decreasing the amount of pressure against which it has to pump. Hypolipidemic agents are not standard management of CHF for pediatric clients. These agents are used to treat hyperlipidemia (elevated cholesterol and triglycerides), which contribute to coronary artery disease. Positive chronotropic agents will increase the heart rate. ➧ Test-taking Tip: Recall that heart failure is inability of the heart to pump enough blood to meet the body's demand for energy. Select options that decrease the workload of the heart and improve cardiac output.

94. When a nurse removes a syringe after giving an intravenous (IV) push medication through a heparin lock port on a central line, the lock port adheres to the syringe and is removed from the central line catheter hub. The nurse notes that the clamp on the central line catheter is still open and suspects that air may have entered the central line catheter. Which immediate actions should be taken by the nurse? SELECT ALL THAT APPLY. 1. Notify the health-care provider 2. Close the clamp on the central line catheter 3. Turn the client onto the right side with the feet higher than the head 4. Attach a 10-mL syringe to the catheter, open the clamp, and aspirate until blood returns 5. Flush the line with 0.9% NaCl and then heparin if Ordered

ANSWER: 1, 2, 4, 5 The health-care provider should be notified; an echocardiogram may be ordered, and the client should be assessed by a physician. The clamp should be closed to prevent additional air from entering the line. Attaching a syringe and aspirating until blood returns may prevent air in the line from entering the central circulation. The line should be flushed with 0.9% NaCl and then heparin, if ordered, to maintain patency of the central line and prevent it from clotting.

90. A nurse is teaching a woman who is 8 weeks pregnant and had a Roux-en-Y gastric bypass surgical procedure 19 months ago. The woman's body mass index (BMI) is now 30.3%, down from 39.5%. Which nutritional deficits associated with gastric bypass should the nurse address with this client? SELECT ALL THAT APPLY. 1. Iron deficiency 2. Vitamin B12 deficiency 3. Vitamin C deficiency 4. Vitamin A deficiency 5. Calcium deficiency 6. Protein deficiency

ANSWER: 1, 2, 4, 5, 6 Reduced intake can result in nutritional deficits. Those most likely to occur after gastric bypass surgery include iron (from malabsorption), vitamin B12 (from reduced intrinsic factor production secondary to parietal cell loss and malabsorption), and vitamin A (from bypassed duodenum where these are primarily absorbed), and calcium and protein (from reduce intake). Vitamin C deficiency has not been associated with Roux-en-Y gastric bypass.

69. A school nurse is educating school-aged children on modifiable risk factors for coronary artery disease (CAD). Which modifiable risk factors should the nurse include in the presentation? SELECT ALL THAT APPLY. 1. Diabetes mellitus 2. Hypertension 3. Age 4. Family history 5. Sedentary lifestyle 6. Obesity

ANSWER: 1, 2, 5, 6 Diabetes mellitus, hypertension, sedentary lifestyle, and obesity are modifiable risk factors for CAD. While age and family are risk factors, they are nonmodifiable.

A school nurse is educating school-aged children on modifiable risk factors for coronary artery disease (CAD). Which modifiable risk factors should the nurse include in the presentation? SELECT ALL THAT APPLY. 1. Diabetes mellitus 2. Hypertension 3. Age 4. Family history 5. Sedentary lifestyle 6. Obesity

ANSWER: 1, 2, 5, 6 Diabetes mellitus, hypertension, sedentary lifestyle, and obesity are modifiable risk factors for CAD. While age and family are risk factors, they are nonmodifiable. ➧ Test-taking Tip:The words "modifiable risk factors" and "CAD" are the key words in the stem.

71. A nurse is taking the health and social history of an adolescent client experiencing episodes of palpitations. Which components of the social history could contribute to the palpitations? SELECT ALL THAT APPLY. 1. Alcohol intake 2. Sexual history 3. Nicotine use 4. Caffeine intake 5. A sports injury to the chest

ANSWER: 1, 3, 4, 5 Alcohol, nicotine, and caffeine could contribute to the palpitations. Nicotine and caffeine are stimulants that increase the heart rate. Although alcohol is a depressant, it has been shown to be linked with supraventricular tachycardia (SVT) and heart palpitations because it irritates the cardiac muscle. Chest trauma during a sports event can induce dysrhythmias. A sexual history is an important part of the social history, although it does not likely contribute to the client experiencing palpitations.

A nurse is taking the health and social history of an adolescent client experiencing episodes of palpitations. Which components of the social history could contribute to the palpitations? SELECT ALL THAT APPLY. 1. Alcohol intake 2. Sexual history 3. Nicotine use 4. Caffeine intake 5. A sports injury to the chest

ANSWER: 1, 3, 4, 5 Alcohol, nicotine, and caffeine could contribute to the palpitations. Nicotine and caffeine are stimulants that increase the heart rate. Although alcohol is a depressant, it has been shown to be linked with supraventricular tachycardia (SVT) and heart palpitations because it irritates the cardiac muscle. Chest trauma during a sports event can induce dysrhythmias. A sexual history is an important part of the social history, although it does not likely contribute to the client experiencing palpitations. ➧ Test-taking Tip: Use knowledge of the physical effects of alcohol, nicotine, caffeine, and trauma. Use the process of elimination to select the correct options.

558. A nurse is planning care for a client admitted with a new diagnosis of persistent atrial fibrillation with rapid ventricular response. Although the client has had no previous cardiac problems, the client has been in atrial fibrillation for more than 2 days. The nurse should anticipate that the health-care provider is likely to initially order: SELECT ALL THAT APPLY. 1. oxygen. 2. immediate cardioversion. 3. administration of amiodarone (Cordarone®). 4. initiation of a IV heparin infusion. 5. immediate catheter-directed ablation of the AV node. 6. administration of a calcium channel antagonist such as diltiazem (Cardizem®).

ANSWER: 1, 3, 4, 6

A client with a diagnosis of chronic obstructive pulmonary disease (COPD) has developed polycythemia vera, and a nurse has completed teaching on measures to prevent complications. During a home health visit, the nurse evaluates that the client is correctly following the teaching when the client: SELECT ALL THAT APPLY. 1. tells the nurse about discontinuing iron supplements. 2. relays increasing alcohol intake to decrease blood viscosity. 3. records the amount consumed after drinking a glass of water. 4. discusses yesterday's phlebotomy treatment to remove blood. 5. shows the nurse a menu plan for eating three large meals daily. 6. reclines in a recliner chair with legs uncrossed, wearing antiembolic stockings (TEDS®).

ANSWER: 1, 3, 4, 6 Iron supplements, including those in multivitamins, should be avoided because the iron stimulates red blood cell production. Increasing fluid intake to 3,000 mL daily will help decrease blood viscosity. Phlebotomy is performed on a routine or intermittent basis to diminish blood viscosity, deplete iron stores, and decrease the client's ability to manufacture excess erythrocytes. Elevating the legs, avoiding constriction or crossing the legs, and wearing antiembolic socks help prevent deep vein thrombosis. Alcohol increases the risk of bleeding. Frequent, small meals are better tolerated, especially if the liver is involved. ➧ Test-taking Tip: Focus on the issue: measures to prevent complications of polycythemia vera.

A nurse is planning care for a client admitted with a new diagnosis of persistent atrial fibrillation with rapid ventricular response. Although the client has had no previous cardiac problems, the client has been in atrial fibrillation for more than 2 days. The nurse should anticipate that the health-care provider is likely to initially order: SELECT ALL THAT APPLY. 1. oxygen. 2. immediate cardioversion. 3. administration of amiodarone (Cordarone®). 4. initiation of a IV heparin infusion. 5. immediate catheter-directed ablation of the AV node. 6. administration of a calcium channel antagonist such as diltiazem (Cardizem®).

ANSWER: 1, 3, 4, 6 The ineffective atrial contractions or loss of atrial kick with atrial fibrillation can decrease cardiac output. Administering oxygen enhances tissue oxygenation. Amiodarone is used for pharmacological cardioversion of the atrial fibrillation rhythm. The client is at risk for thrombi in the atria from stasis. Anticoagulant therapy is used to prevent thromboembolism. Diltiazem, a calcium channel antagonist, is prescribed to slow the ventricular response to atrial fibrillation. An alternative to a calcium channel antagonist would be the use of a beta blocker, such as esmolol, metoprolol, or propranolol. Cardioversion would only be considered if medications were ineffective in converting the client's rhythm and only after the presence of an atrial clot has been ruled out. Ablation of the AV node would only be considered if medications were ineffective in controlling the client's heart rate. ➧ Test-taking Tip: Carefully read the information provided in the stem. The key phrase is "initially order." The nurse should direct interventions at the client's potential complications from the arrhythmia. Note that both options 2 and 5 contain the words "immediate." Eliminate one or both of those options, because both procedures cannot be immediate.

A nurse is planning care for a client admitted with a new diagnosis of persistent atrial fibrillation with rapid ventricular response. Although the client has had no previous cardiac problems, the client has been in atrial fibrillation for more than 2 days. The nurse should anticipate that the health-care provider is likely to initially order: SELECT ALL THAT APPLY. 1. oxygen. 2. immediate cardioversion. 3. administration of amiodarone (Cordarone®). 4. initiation of a IV heparin infusion. 5. immediate catheter-directed ablation of the AV node. 6. administration of a calcium channel antagonist such as diltiazem (Cardizem®).

ANSWER: 1, 3, 4, 6 The ineffective atrial contractions or loss of atrial kick with atrial fibrillation can decrease cardiac output. Administering oxygen enhances tissue oxygenation. Amiodarone is used for pharmacological cardioversion of the atrial fibrillation rhythm. The client is at risk for thrombi in the atria from stasis. Anticoagulant therapy is used to prevent thromboembolism. Diltiazem, a calcium channel antagonist, is prescribed to slow the ventricular response to atrial fibrillation. An alternative to a calcium channel antagonist would be the use of a beta blocker, such as esmolol, metoprolol, or propranolol. Cardioversion would only be considered if medications were ineffective in converting the client's rhythm and only after the presence of an atrial clot has been ruled out. Ablation of the AV node would only be considered if medications were ineffective in controlling the client's heart rate.

88. A nurse is assessing a 76-year-old female client. Which findings require physician consultation? SELECT ALL THAT APPLY. 1. 2+ pitting edema of lower extremities 2. Increased hair thinning 3.Warty growth on the vulva 4. Presence of actinic lentigo 5. Port-wine angioma on the arm 6. 6-mm asymmetric dark lesion on the perineum

ANSWER: 1, 3, 6 Pitting edema can be a sign of fluid volume overload from cardiac, renal, endocrine, medication-related, or another problem. The warty growth on the vulva could be a sign of human papillomavirus (HPV) infection. The 6-mm asymmetric dark lesion is a hallmark sign for cancer. Increased hair thinning and actinic lentigo (sun freckles or age spots) are normal findings in the older adult population. Port-wine angioma is a benign vascular tumor that involves the skin and the subcutaneous tissues that are present at birth, usually persisting indefinitely.

A nurse is caring for a client with renal insufficiency. In addition to an ordered fluid restriction, the client needs strict monitoring of intake and output. Which actions should the nurse plan to include when caring for the client? SELECT ALL THAT APPLY. 1. Discussing with the client and family the plan of care and fluid restriction 2. Documenting pureed foods as part of the client's liquid intake 3. Eliminating counting ice chips as intake because this represents such a small amount of intake 4. Providing a collection device for measuring the client's urine output 5. Instructing the family to record any intake they provide to the client on the facility intake record 6. Encouraging the family to bring favorite food items from home for the client to eat

ANSWER: 1, 4 Informing the client and family in the plan of care helps to provide reinforcement for the client and to ensure compliance with the fluid restriction and plan. Measurement and collection devices are necessary and beneficial when strict monitoring is required. Pureed foods are not counted as liquid because they are considered solid in a different form. Ice chips are considered fluid; a 200 mL cup of ice is equal to 100 mL of water. Only health care personnel should document on official agency records. The family should be informed to not provide the client with addition liquid intake. Renal insufficiency will warrant food and fluid restrictions. Bringing favorite food items from home should be discouraged to ensure that the client follows the plan of care for fluid and electrolyte restrictions. ➧ Test-taking Tip: Think about the food and fluid restrictions that are likely with renal insufficiency and measures that the nurse can use to ensure that the client adheres to the plan of care.

student nurse is assisting a registered nurse (RN) in the care of a postpartum client who is 48 hours post-vaginal delivery. The student reports finding a warm, red, tender area on the client's left calf. The nurse assesses the client and explains to the student that postpartum clients are at increased risk for thrombophlebitis because of which of the following? SELECT ALL THAT APPLY. 1. The fibrinogen levels in the blood are elevated. 2. Fluids normally shift from the interstitial to the intravascular space. 3. Postpartum hormonal shifts irritate vascular basement membranes. 4. The legs are elevated in stirrups at the time of delivery. 5. Dilation of the veins in the lower extremities is present. 6. Compression of the common iliac vein occurs during pregnancy.

ANSWER: 1, 4, 5, 6 Major causes of thromboembolic disease are hypercoagulability of the blood and venous stasis. During pregnancy, fibrinogen levels increase, and this increase continues to be present in the postpartum period. Elevation of the legs in stirrups during delivery leads to pooling of blood and vascular stasis. Dilation of the veins in the lower extremities occurs during pregnancy as does compression of the common iliac vein. Both of these pregnancy changes increase the risk of venous stasis. There is not a shift of fluid from the interstitial to the vascular spaces in the postpartum period. Actual blood volume increases during pregnancy and is further increased immediately after delivery. This fluid volume is eventually lost through diuresis during the first postpartum week. Postpartum hormonal changes do occur but they do not affect the vascular basement membranes

A nurse receives a serum laboratory report for six different clients with admitting diagnoses of chest pain. After reviewing all of the lab reports, in which order should the nurse address each lab value? Prioritize the order in which the nurse should address each of the clients' results. ______ Troponin T 42 ng/mL (0.0-0.4 ng/mL) ______ WBC 11,000 K/μL ______ Hgb 7.2 g/dL ______ SCr 2.2 mg/dL ______ K 2.2 mEq/L ______ Total cholesterol 430 mg/dL

ANSWER: 1, 6, 3, 4, 2, 5 The nurse should address the elevated troponin level first. Cardiospecific troponins (troponin T, cTnT, and troponin I, cTnI) are released into circulation after myocardial injury and are highly specific indicators of myocardial infarction. Since "time is muscle," the client needs to be treated immediately to prevent extension of the infarct and possible death. The nurse should address the decreased serum potassium level (K) second. The normal serum K level is 3.5 to 5.8 mEq/L. A low serum K level can cause life-threatening dysrhythmias. The normal hemoglobin (Hgb) is 13.1 to 17.1 g/dL. A low Hgb can contribute to inadequate tissue perfusion and contribute to myocardial ischemia. The normal serum creatinine (SCr) is 0.4 to 1.4 mg/dL. Impaired circulation may be causing this alteration and further client assessment is needed. Medication doses may need to be adjusted with impaired renal perfusion. The normal total serum cholesterol should be less than 200 mg/dL. This is a risk factor for development of coronary artery disease. The client needs teaching. The normal white blood cell (WBC) count is 3.9 to 11.9 K/μL. Because the finding is normal, it can be addressed last. ➧ Test-taking Tip: Knowledge of the normal ranges and the significance for critical laboratory values is expected on the NCLEX-RN®. Use the ABCs (airway, breathing, circulation) to determine priority. Of the laboratory values, determine those that are related to tissue perfusion (circulation), and then determine which value is most life threatening.

A nurse is evaluating the blood pressure (BP) results for multiple clients with cardiac problems on a telemetry unit. Which BP reading suggests to the nurse that the client's mean arterial pressure (MAP) is abnormal and warrants notifying the physician? 1. 94/60 mm Hg 2. 98/36 mm Hg 3. 110/50 mm Hg 4. 140/78 mm Hg .

ANSWER: 2 A MAP of less than 60 mm Hg indicates that there is inadequate perfusion to organs. The mean arterial pressure is calculated by the sum of the SBP + 2DBP and then divided by 3 [MAP = (SBP + 2DBP)/3]. Thus the MAP of 98/36 mm Hg is (98 + 72)/3 = 170/3 = 56.7. The mean arterial pressure of 94/60 is 71.3. The mean arterial pressure of 110/50 is 70. The mean arterial pressure of 140/78 is 98.7. ➧ Test-taking Tip: Focus on the issue of the question, a BP reading with a MAP of less than 60. Though a BP of 94/60 mm Hg and 140/78 mm Hg may warrant notifying the physician, the question is asking for a BP with an abnormal MAP (less than 70). Normal MAP is 70 to 100

A client is admitted to a coronary care unit following an anterior myocardial infarction (MI). A nurse, caring for the client, obtains the following assessment findings. Based on these findings, the nurse should immediately notify the physician and plan which intervention? TIME 7:00 8:00 8:30 9:00 Temp Tm=Tympanic O=Oral 98.6 R=Rectal Ax=Axillary (0) Pulse 96 104 118 Resp 22 28 32 BP 116 102 85 mm Hg 80 56 48 POx O2 L/M/NC 2 4 Pain Rating: Numeric 0-10 0 4 INTAKE AND OUTPUT 7:00 8:00 8:30 9:00 9:30 IVPB Oral Intake Urine Output 30 mL 20 mL 1. Administering an IV fluid bolus of 0.9% NaCl because the client is in right heart failure 2. Initiating an IV infusion of dopamine (Intropin®) because the client is in cardiogenic shock 3. Preparing the client for pericardiocentesis since cardiac tamponade is suspected 4. Calling for a stat chest x-ray to rule out pulmonary embolism (PE)

ANSWER: 2 A complication of a anterior MI is left ventricular failure because a large portion of the left ventricle may have been damaged. Damage to the left ventricle leads to reduced cardiac output and cardiogenic shock. Hypotension, tachycardia, tachypnea, and decreasing urine output are classic signs of cardiogenic shock. A client with a systolic blood pressure (SBP) 85 mm Hg and a diastolic blood pressure (DBP) of 40 mm Hg has a mean arterial pressure (MAP) of 55. (MAP = [(SBP) + (2DBP)]/3 or 85 + 40 + 40 = 165/3 = 55). A MAP of less than 60 indicates impaired perfusion to organs. Vasopressors and positive inotropes are administered in cardiogenic shock to increase cardiac output. IV fluids may be administered if the client is showing signs of dehydration. However, the client is not experiencing right-sided heart failure (RHF). RHF produces signs of venous congestion, including jugular venous distention, hepatomegaly, splenomegaly, and ascites. Although symptoms of cardiac tamponade do include hypotension, tachycardia, and tachypnea, other signs include muffled heart sounds. Appropriate treatment for cardiac tamponade is pericardiocentesis. A chest x-ray is used to diagnose PE, but the data do not suggest PE. The signs of PE can be subtle and nonspecific but most commonly present with dyspnea, tachycardia, tachypnea, and/or chest pain. ➧ Test-taking Tip: Focus on the client vital sign changes and decreased urine output. Complications of an anterior MI include left ventricular failure, reduced cardiac output, and cardiogenic shock.

A nurse increases activity for a client with an admitting diagnosis of acute coronary syndrome. Which symptoms experienced by the client best support a nursing diagnosis of activity intolerance? 1. Pulse rate increased by 15 beats per minute during activity 2. Blood pressure (BP) 130/86 mm Hg before activity; BP 108/66 mm Hg during activity 3. Increased dyspnea and diaphoresis relieved when sitting in a chair 4. A mean arterial pressure (MAP) of 80 following activity .

ANSWER: 2 A drop in BP of 20 mm Hg from the baseline indicates that the client's heart is unable to adapt to the increased energy and oxygen demands of the activity. An increased heart rate during activity and the relief of dyspnea and diaphoresis with rest indicates the heart is able to adapt. A MAP of 80 is normal. ➧ Test-taking Tip:The key words are "best supports." Select the option that is an abnormal finding

A nurse increases activity for a client with an admitting diagnosis of acute coronary syndrome. Which symptoms experienced by the client best support a nursing diagnosis of activity intolerance? 1. Pulse rate increased by 15 beats per minute during activity 2. Blood pressure (BP) 130/86 mm Hg before activity; BP 108/66 mm Hg during activity 3. Increased dyspnea and diaphoresis relieved when sitting in a chair 4. A mean arterial pressure (MAP) of 80 following Activity

ANSWER: 2 A drop in BP of 20 mm Hg from the baseline indicates that the client's heart is unable to adapt to the increased energy and oxygen demands of the activity. An increased heart rate during activity and the relief of dyspnea and diaphoresis with rest indicates the heart is able to adapt. A MAP of 80 is normal. ➧ Test-taking Tip:The key words are "best supports." Select the option that is an abnormal finding

75. A nurse establishes a nursing diagnosis of Risk for excess fluid volume for a client diagnosed with heart failure. Which physiological change resulting from heart failure supports this diagnosis? 1. Increased glomerular filtration rate (GFR) 2. Increased antidiuretic hormone (ADH) production 3. Increased sodium excretion 4. Increased cardiac output

ANSWER: 2 ADH is produced in response to changes in intravascular volume. The result is increased water reabsorption. A decrease in GFR would not put the client at risk for excess fluid volume. Increased sodium excretion usually results in increased fluid output and would not place the client at risk for excess fluid volume. Increased cardiac output usually increases perfusion to the kidney, resulting in increased output, and does not place the client at risk for excess fluid volume.

A nurse establishes a nursing diagnosis of Risk for excess fluid volume for a client diagnosed with heart failure. Which physiological change resulting from heart failure supports this diagnosis? 1. Increased glomerular filtration rate (GFR) 2. Increased antidiuretic hormone (ADH) production 3. Increased sodium excretion 4. Increased cardiac output

ANSWER: 2 ADH is produced in response to changes in intravascular volume. The result is increased water reabsorption. A decrease in GFR would not put the client at risk for excess fluid volume. Increased sodium excretion usually results in increased fluid output and would not place the client at risk for excess fluid volume. Increased cardiac output usually increases perfusion to the kidney, resulting in increased output, and does not place the client at risk for excess fluid volume. ➧ Test-taking Tip: Read each option carefully to determine if it would increase or decrease fluid volume.

A 33-year-old client reports left leg pain, right-sided chest pain, and a sudden onset of shortness of breath. Which action should be taken immediately by the nurse? 1. Take the client's temperature 2. Auscultate the client's the lung sounds 3. Percuss the client's abdomen 4. Request a stat chest x-ray ANSWER: 2 Auscultation of lung sounds should be one of the first assessments performed by the nurse to determine the cause of the client's shortness of breath. Chest x-ray would be helpful in assessing the cause of shortness of breath but would need a physician's order and would not be the first priority. Percussion of abdomen and measurement of the client's temperature are helpful tools when completing a full assessment but are not priority in this situation. ➧ Test-taking Tip: Think about the ABCs: airway, breathing, circulation. Airway is priority

ANSWER: 2 Auscultation of lung sounds should be one of the first assessments performed by the nurse to determine the cause of the client's shortness of breath. Chest x-ray would be helpful in assessing the cause of shortness of breath but would need a physician's order and would not be the first priority. Percussion of abdomen and measurement of the client's temperature are helpful tools when completing a full assessment but are not priority in this situation. ➧ Test-taking Tip: Think about the ABCs: airway, breathing, circulation. Airway is priority

Following a normal chest x-ray for a client who had cardiac surgery, a nurse receives an order to remove the chest tubes. Which intervention should the nurse plan to implement first? 1. Auscultate the client's lung sounds 2. Administer 4 mg morphine sulfate intravenously 3. Turn off the suction to the chest drainage system 4. Prepare the dressing supplies at the client's bedside

ANSWER: 2 Because the peak action of morphine sulfate is 10 to 15 minutes, this should be administered first. Auscultating the client's lungs before and after the procedure, turning off the suction, and assembling the dressing supplies are all necessary, but administering the analgesic should be first.

Following a normal chest x-ray for a client who had cardiac surgery, a nurse receives an order to remove the chest tubes. Which intervention should the nurse plan to implement first? 1. Auscultate the client's lung sounds 2. Administer 4 mg morphine sulfate intravenously 3. Turn off the suction to the chest drainage system 4. Prepare the dressing supplies at the client's bedside

ANSWER: 2 Because the peak action of morphine sulfate is 10 to 15 minutes, this should be administered first. Auscultating the client's lungs before and after the procedure, turning off the suction, and assembling the dressing supplies are all necessary, but administering the analgesic should be first. ➧ Test-taking Tip: Recall that focusing on the client should be the priority.

A nurse admits a client who is 28 weeks pregnant and experiencing congestive heart failure. When initiating a health-care provider's admission orders for the client, which order should the nurse question? 1. Furosemide (Lasix®) 40 mg IV bid 2. Captopril (Capoten®) 25 mg PO daily 3. Digoxin (Lanoxin®) 0.125 mg IV daily 4. Metoprolol sustained release (Toprol XL®) 50 mg PO daily

ANSWER: 2 Captopril is an angiotensin-converting enzyme (ACE) inhibitor. ACE inhibitors are contraindicated in the second and third trimesters of pregnancy. They can cause oligohydramnios, intrauterine growth retardation (IUGR), congenital structural defects, and renal failure. Digoxin, furosemide, and metoprolol are all Category C medications, but they have not been shown to cause fetal harm. Digoxin exerts a positive inotropic action on the heart. Metoprolol, which is a beta blocker, can improve left ventricular ejection fraction and slow the progression of heart failure, and furosemide is a diuretic.

A client is transferred to a medical center from a local hospital and undergoes emergency treatment related to acute coronary syndrome. The client has slept little over the past 24 hours. Which assessment finding is consistent with sleep deprivation? 1. Periods of apnea lasting greater than 5 seconds 2. Slowing of thought processes 3. Hyperventilation 4. Cool extremities

ANSWER: 2 Clients who become sleep-deprived often show signs of impaired cognitive functioning. The other assessment findings are not associated with sleep deprivation. ➧ Test-taking Tip: Focus on the issue: sleep deprivation. Evaluate each option and select option 2 because it relates to brain function.

63. In reviewing a physician's orders for a postoperative client who underwent gynecological surgery, which order should a nurse determine is specifically written with the intent to prevent postoperative thrombophlebitis and pulmonary embolism? 1. Have the client dangle the legs the evening of surgery 2. Administer enoxaparin (Lovenox®) 40 mg subcutaneously daily 3. Administer hydromorphone (Dilaudid®) 1 to 4 mg IV every 3 to 4 hours as needed (prn) 4. Encourage coughing and deep breathing (C&DB) every hour while awake

ANSWER: 2 Enoxaparin is an anticoagulant that potentiates the inhibitory effect of antithrombin on factor Xa and thrombin. Early postoperative ambulation instead of dangling is a major preventive technique for thrombophlebitis. Hydromorphone is a narcotic analgesic for pain control. Coughing and deep breathing promote lung expansion and prevent atelectasis and pneumonia. ➧ Test-taking Tip: Note the key words "specifically written," and then eliminate options 1, 3, and 4 because they are not specific to preventing postoperative thrombophlebitis and pulmonary embolism. Knowledge of medications is needed to answer this question

In reviewing a physician's orders for a postoperative client who underwent gynecological surgery, which order should a nurse determine is specifically written with the intent to prevent postoperative thrombophlebitis and pulmonary embolism? 1. Have the client dangle the legs the evening of surgery 2. Administer enoxaparin (Lovenox®) 40 mg subcutaneously daily 3. Administer hydromorphone (Dilaudid®) 1 to 4 mg IV every 3 to 4 hours as needed (prn) 4. Encourage coughing and deep breathing (C&DB) every hour while awake

ANSWER: 2 Enoxaparin is an anticoagulant that potentiates the inhibitory effect of antithrombin on factor Xa and thrombin. Early postoperative ambulation instead of dangling is a major preventive technique for thrombophlebitis. Hydromorphone is a narcotic analgesic for pain control. Coughing and deep breathing promote lung expansion and prevent atelectasis and pneumonia. ➧ Test-taking Tip: Note the key words "specifically written," and then eliminate options 1, 3, and 4 because they are not specific to preventing postoperative thrombophlebitis and pulmonary embolism. Knowledge of medications is needed to answer this question.

70. The parents of a pediatric client report that their child is experiencing palpitations, dizziness, diaphoresis, and chest pain. The client is diagnosed with supraventricular tachycardia (SVT). A nurse instructs the parents on techniques to reverse future episodes of SVT. Which technique stated by a parent indicates further teaching is needed? 1. Wrap the child's head with a cold, wet towel. 2. Massage the child's carotid arteries bilaterally. 3. Have the child perform the Valsalva's maneuver. 4. Administer medications after taking the child's pulse for 1 full minute.

ANSWER: 2 Further teaching is needed when a parent states to bilaterally massage the carotid arteries. This should not be employed to reverse SVT because it may restrict blood flow. Massaging the carotid arteries unilaterally, however, is a technique used to convert SVT and should be included in the teaching plan. Wrapping the child's head with a cold, wet towel, having the child perform the Valsalva's maneuver (bearing down), and taking the pulse prior to administering medications are techniques that the nurse should teach the parents to use when symptoms of SVT are present because these can potentially convert the SVT rhythm to a sinus rhythm.

The parents of a pediatric client report that their child is experiencing palpitations, dizziness, diaphoresis, and chest pain. The client is diagnosed with supraventricular tachycardia (SVT). A nurse instructs the parents on techniques to reverse future episodes of SVT. Which technique stated by a parent indicates further teaching is needed? 1. Wrap the child's head with a cold, wet towel. 2. Massage the child's carotid arteries bilaterally. 3. Have the child perform the Valsalva's maneuver. 4. Administer medications after taking the child's pulse for 1 full minute.

ANSWER: 2 Further teaching is needed when a parent states to bilaterally massage the carotid arteries. This should not be employed to reverse SVT because it may restrict blood flow. Massaging the carotid arteries unilaterally, however, is a technique used to convert SVT and should be included in the teaching plan. Wrapping the child's head with a cold, wet towel, having the child perform the Valsalva's maneuver (bearing down), and taking the pulse prior to administering medications are techniques that the nurse should teach the parents to use when symptoms of SVT are present because these can potentially convert the SVT rhythm to a sinus rhythm. ➧ Test-taking Tip: Read each option carefully. Think about how each option will affect the SVT rhythm as well as blood flow to the brain.

56. A client diagnosed with von Willebrand's disease calls a clinic after experiencing hemarthrosis. Which treatment should a nurse recommend? 1. "Treat the pain with two 325-mg aspirin (Ecotrin®) tablets every 4 hours." 2. "Apply cold packs 2 hours on and 2 hours off of the affected site for 24 to 48 hours." 3. "Come to the clinic immediately so you can receive an infusion of fresh frozen plasma." 4. "If you are wearing a splint, remove it immediately to avoid compartment syndrome."

ANSWER: 2 Hemarthrosis is bleeding into the joint. The pressure of the ice pack and cold will reduce the bleeding and swelling. Aspirin and NSAIDs are contraindicated because they interfere with platelet aggregation. The client and family are usually taught how to administer factor concentrates at home at the first sign of bleeding. The splint should be left on initially to control bleeding. The client should be instructed on how to assess for adequate tissue perfusion.

client diagnosed with von Willebrand's disease calls a clinic after experiencing hemarthrosis. Which treatment should a nurse recommend? 1. "Treat the pain with two 325-mg aspirin (Ecotrin®) tablets every 4 hours." 2. "Apply cold packs 2 hours on and 2 hours off of the affected site for 24 to 48 hours." 3. "Come to the clinic immediately so you can receive an infusion of fresh frozen plasma." 4. "If you are wearing a splint, remove it immediately to avoid compartment syndrome."

ANSWER: 2 Hemarthrosis is bleeding into the joint. The pressure of the ice pack and cold will reduce the bleeding and swelling. Aspirin and NSAIDs are contraindicated because they interfere with platelet aggregation. The client and family are usually taught how to administer factor concentrates at home at the first sign of bleeding. The splint should be left on initially to control bleeding. The client should be instructed on how to assess for adequate tissue perfusion. ➧ Test-taking Tip: If unsure of the meaning of hemarthrosis, apply knowledge of medical terminology. "Heme-" refers to blood, and "arthrosis" refers to joints. Remember that cold reduces bleeding.

Elevated homocysteine levels are associated with the development of arteriosclerosis and venous thrombosis. A clinic nurse should teach a client that the dietary therapy to decrease homocysteine levels includes eating foods rich in: 1. monosaturated fats. 2. B-complex vitamins. 3. vitamin C. 4. calcium.

ANSWER: 2 Homocysteine interferes with the elasticity of the endothelial layer in blood vessels. Foods rich in B-complex vitamins, especially folic acid, have been found to lower serum homocysteine levels. Monosaturated fats, vitamin C, and calcium are included in a healthy diet but have not been found to affect the homocysteine levels. ➧ Test-taking Tip: Key words in the stem are "decrease homocysteine." The question is "What foods would lower high levels of homocysteine?"

A client experiences cardiac arrest at home and is successfully resuscitated. Following placement of an implantable cardioverter-defibrillator (ICD), a nurse is evaluating the effectiveness of teaching for the client. Which statement, if made by the client, indicates that further teaching is needed? 1. "The ICD will monitor my heart activity and provide a shock to my heart if my heart goes into ventricular fibrillation again." 2. "When I feel the first shock I should tell my family to start cardiopulmonary resuscitation (CPR) and call 911." 3. "I am fearful of my first shock since my friend stated his shock felt like a blow to the chest." 4. "I will need to ask my physician when I can resume driving because some states disallow driving until there is a 6-month discharge-free period."

ANSWER: 2 If the first shock is unsuccessful, the device will recycle and continue to deliver shocks. If the device fires more than once, the emergency medical services (EMS) system should be activated. The ICD continues to deliver shocks if indicated; CPR should only be initiated after the shocks have been delivered and only if the client is unresponsive and pulseless. The ICD monitors the client's heart rate and rhythm, identifies ventricular tachycardia or ventricular fibrillation, and delivers a 25-joule or less shock if a lethal rhythm is detected. Various sensations have been described when the device delivers a shock, including a blow to or kick in the chest. State laws vary regarding drivers with ICDs. The decision regarding driving is also based on whether dysrhythmias are present, the frequency of firing, and the client's overall health. ➧ Test-taking Tip:The key phrase "further teaching is needed" indicate a false-response item. Select the client's statement that is not correct.

A client experiences cardiac arrest at home and is successfully resuscitated. Following placement of an implantable cardioverter-defibrillator (ICD), a nurse is evaluating the effectiveness of teaching for the client. Which statement, if made by the client, indicates that further teaching is needed? 1. "The ICD will monitor my heart activity and provide a shock to my heart if my heart goes into ventricular fibrillation again." 2. "When I feel the first shock I should tell my family to start cardiopulmonary resuscitation (CPR) and call 911." 3. "I am fearful of my first shock since my friend stated his shock felt like a blow to the chest." 4. "I will need to ask my physician when I can resume driving because some states disallow driving until there is a 6-month discharge-free period."

ANSWER: 2 If the first shock is unsuccessful, the device will recycle and continue to deliver shocks. If the device fires more than once, the emergency medical services (EMS) system should be activated. The ICD continues to deliver shocks if indicated; CPR should only be initiated after the shocks have been delivered and only if the client is unresponsive and pulseless. The ICD monitors the client's heart rate and rhythm, identifies ventricular tachycardia or ventricular fibrillation, and delivers a 25-joule or less shock if a lethal rhythm is detected. Various sensations have been described when the device delivers a shock, including a blow to or kick in the chest. State laws vary regarding drivers with ICDs. The decision regarding driving is also based on whether dysrhythmias are present, the frequency of firing, and the client's overall health.

A nurse assesses the pain level of a Native American pediatric client recovering from cardiac surgery. Knowing that Native American pediatric clients may not express pain, the nurse reviews the child's pulse and blood pressure readings following analgesic administration. Which finding should indicate to the nurse that the client's pain is not well-controlled? 1. Decreased heart rate and decreased blood pressure 2. Increased heart rate and increased blood pressure 3. Increased heart rate and decreased blood pressure 4. Decreased heart rate and increased blood pressure

ANSWER: 2 Increased heart rate and blood pressure may be indicative of postoperative pain in a pediatric client. A decreased heart rate and blood pressure could indicate that analgesics are effective for pain control. An increased heart rate and decreased blood pressure could be signs of bleeding. A decreased heart rate and increased blood pressure could be a sign of a neurological complication associated with cardiac surgery. ➧ Test-taking Tip: Note that options 1 and 2 are opposites and 3 and 4 are opposites. First examine the options that are opposites and eliminate one or both of these.

A nurse is caring for a client immediately following insertion of a permanent pacemaker via the right subclavian vein approach. The nurse best prevents pacemaker lead dislodgement by: 1. inspecting the incision site dressing for bleeding and the incision for approximation. 2. limiting the client's right arm activity and preventing the client reaching above shoulder level. 3. assisting the client with getting out of bed and ambulating with a walker. 4. ordering a stat chest x-ray following return from the implant procedure.

ANSWER: 2 Limiting arm and shoulder activity initially and up to 24 hours after the pacing leads are implanted helps prevent lead dislodgement. Often an arm sling is used as a reminder to the client to limit arm activity. The dressing should not be removed to check the incision immediately after insertion but should be checked for bleeding to monitor for potential complications. The nurse should assist the client the first time out of bed following a pacemaker implant, but the client should not use a walker for 24 hours after the procedure, and out of bed activity would not resume until the client is stable. A postinsertion chest x-ray is done to check lead placement and to rule out a pneumothorax. It does not promote the intactness of pacing leads. ➧ Test-taking Tip: Focus on the issue of the question: measures to promote intactness of the pacing leads. Analyze the options to determine which would impact the desired result, maintaining intactness of the pacing leads.

A nurse is caring for a client immediately following insertion of a permanent pacemaker via the right subclavian vein approach. The nurse best prevents pacemaker lead dislodgement by: 1. inspecting the incision site dressing for bleeding and the incision for approximation. 2. limiting the client's right arm activity and preventing the client reaching above shoulder level. 3. assisting the client with getting out of bed and ambulating with a walker. 4. ordering a stat chest x-ray following return from the implant procedure.

ANSWER: 2 Limiting arm and shoulder activity initially and up to 24 hours after the pacing leads are implanted helps prevent lead dislodgement. Often an arm sling is used as a reminder to the client to limit arm activity. The dressing should not be removed to check the incision immediately after insertion but should be checked for bleeding to monitor for potential complications. The nurse should assist the client the first time out of bed following a pacemaker implant, but the client should not use a walker for 24 hours after the procedure, and out of bed activity would not resume until the client is stable. A postinsertion chest x-ray is done to check lead placement and to rule out a pneumothorax. It does not promote the intactness of pacing leads. ➧ Test-taking Tip: Focus on the issue of the question: measures to promote intactness of the pacing leads. Analyze the options to determine which would impact the desired result, maintaining intactness of the pacing leads.

A nurse evaluates that a client recognizes foods that are high in calcium when the client selects: 1. 1 cup whole milk, 1 cup spinach, and 3 ounces sardines. 2. 1 cup low-fat yogurt, 1 cup broccoli, and 3 ounces sardines. 3. 1⁄2 cup 2% cottage cheese, 1 cup spinach, and 3 ounces frozen tofu. 4. 1 medium baked potato with 1 tbsp fat-free sour cream, 1 cup spinach, and 3 ounces tofu.

ANSWER: 2 One cup of low-fat plain yogurt has 448 mg of calcium, 1 cup of broccoli has 60 mg, and 3 ounces of sardines have 324 mg, for a total of 832 mg of calcium. One cup of whole milk has 300 mg of calcium, 1 cup of spinach has 30 mg calcium, and 3 ounces of sardines have 324 mg, for a total of 654 mg. A half cup of 2% cottage cheese has 78 mg calcium, 1 cup of spinach has 30 mg, and 3 ounces tofu has 310 mg, for a total of 418 mg. A medium baked potato has 38 mg of calcium, 1 tbsp of low-fat sour cream has 20 mg, 1 cup of spinach has 30 mg, and 3 ounces tofu has 310 mg, for a total of 398 mg calcium. ➧ Test-taking Tip: Use the process of elimination, noting that options 1 and 2 both contain sardines and 3 and 4 both contain tofu. Next, look at the dairy content in each option. Eliminate options 3 and 4 because the volume of dairy content is low. Select option 2, knowing that low-fat plain yogurt has more calcium than whole milk.

A client admitted with a diagnosis of acute coronary syndrome calls for a nurse after experiencing sharp chest pains that radiate to the left shoulder. The nurse notes, prior to entering the client's room, that the client's rhythm is sinus tachycardia with a 10-beat run of premature ventricular contractions (PVCs). Admitting orders included all of the following interventions for treating chest pain. Which should the nurse implement first? 1. Obtain a stat 12-lead electrocardiogram (ECG). 2. Administer oxygen by nasal cannula. 3. Administer sublingual nitroglycerin. 4. Administer morphine sulfate intravenously.

ANSWER: 2 Oxygen should be available in the room and should be initiated first to enhance oxygen flow to the myocardium. Though a stat 12-lead ECG is needed to identify ischemia or infarct location, the first action is to treat the client. Sublingual nitroglycerin dilates coronary arteries and will enhance blood flow to the myocardium. Once oxygen is in place and the vital signs known, nitroglycerin should be administered. Morphine sulfate is a narcotic analgesic used for pain control and anxiety reduction. Because it is a controlled substance, extra steps are needed to retrieve the medication from a secure source, so this is not the first action. ➧ Test-taking Tip: Use the ABCs (airway, breathing, circulation) to establish the priority action. Improving oxygen flow to the myocardium is priority.

A client admitted with a diagnosis of acute coronary syndrome calls for a nurse after experiencing sharp chest pains that radiate to the left shoulder. The nurse notes, prior to entering the client's room, that the client's rhythm is sinus tachycardia with a 10-beat run of premature ventricular contractions (PVCs). Admitting orders included all of the following interventions for treating chest pain. Which should the nurse implement first? 1. Obtain a stat 12-lead electrocardiogram (ECG). 2. Administer oxygen by nasal cannula. 3. Administer sublingual nitroglycerin. 4. Administer morphine sulfate intravenously.

ANSWER: 2 Oxygen should be available in the room and should be initiated first to enhance oxygen flow to the myocardium. Though a stat 12-lead ECG is needed to identify ischemia or infarct location, the first action is to treat the client. Sublingual nitroglycerin dilates coronary arteries and will enhance blood flow to the myocardium. Once oxygen is in place and the vital signs known, nitroglycerin should be administered. Morphine sulfate is a narcotic analgesic used for pain control and anxiety reduction. Because it is a controlled substance, extra steps are needed to retrieve the medication from a secure source, so this is not the first action. ➧ Test-taking Tip: Use the ABCs (airway, breathing, circulation) to establish the priority action. Improving oxygen flow to the myocardium is priority.

74. A 65-year-old client with a history of coronary artery disease is admitted with fluid volume overload. Bumetanide (Bumex®) is administered, and the client's serum potassium level drops to 3.0 mEq/L; intravenous (IV) potassium replacement is ordered. Which factor should a nurse consider when preparing to administer the IV potassium replacement? 1. The potassium concentration should not exceed 20 mEq/L. 2. Ice or warm packs may be needed to reduce vein irritation. 3. The potassium should be administered IV push. 4. The potassium should be added to the IV solution that is infusing.

ANSWER: 2 Potassium can be irritating to the vein, and the client may complain of burning. Strategies to minimize pain and inflammation include ice or warm packs. Although the usual replacement dose is 20 mEq/100 mL with administration of 10 to 20 mEq/hr, concentrations can safely range from 10 to 40 mEq/L. Potassium is never administered as an IV push; it will cause cardiac dysrhythmias. Adding medication to an already-infusing IV solution is unsafe and can result in a faster or slower rate of administration, depending on the volume of solution remaining.

A 65-year-old client with a history of coronary artery disease is admitted with fluid volume overload. Bumetanide (Bumex®) is administered, and the client's serum potassium level drops to 3.0 mEq/L; intravenous (IV) potassium replacement is ordered. Which factor should a nurse consider when preparing to administer the IV potassium replacement? 1. The potassium concentration should not exceed 20 mEq/L. 2. Ice or warm packs may be needed to reduce vein irritation. 3. The potassium should be administered IV push. 4. The potassium should be added to the IV solution that is infusing.

ANSWER: 2 Potassium can be irritating to the vein, and the client may complain of burning. Strategies to minimize pain and inflammation include ice or warm packs. Although the usual replacement dose is 20 mEq/100 mL with administration of 10 to 20 mEq/hr, concentrations can safely range from 10 to 40 mEq/L. Potassium is never administered as an IV push; it will cause cardiac dysrhythmias. Adding medication to an already-infusing IV solution is unsafe and can result in a faster or slower rate of administration, depending on the volume of solution remaining. ➧ Test-taking Tip: Note that options 3 and 4 both address methods of administration. Because both cannot be correct, either one or both are incorrect.

65. A nurse is reviewing the chart of a client with a diagnosis of stage II heart failure (see abbreviated chart). The data should suggest to the nurse that: Admitting History & Physical Serum Laboratory Data Diagnostic Data Results Medications • Experiencing dyspnea on • exertion • Lung sounds: • crackles in bilateral bases• Reports seeing yellow halo when visualizing objects Physical Serum Laboratory BNP: 886 ng/L* exertion • aPTT: 55 sec** K+: 2.9 mEq/L *B-natriuretic peptide (BNP) normal value 100 ng/L **Activated partial thromboplastin time (aPTT) 55 sec normal range 24 to 33 seconds Diagnostic Data Results • 12-lead ECG results: Atrial fibrillation with frequent PVCs. • Chest x-ray: Pulmonary infiltrated bilateral bases. Cardiomegaly Medications • Buffered aspirin 325 mg oral daily • Digoxin (Lanoxin®) 0.25 mg oral daily • Furosemide (Lasix®) 40 mg oral daily • Atenolol (Tenormin®) 25 mg oral • Heparin intravenous infusion per protocol 1. medications should be administered as ordered. 2. the client may be experiencing digoxin toxicity. 3. hyperkalemia likely caused the client's cardiac dysrhythmias. 4. the client's visual disturbance likely resulted from the atrial fibrillation rhythm or effects of the anticoagulants.

ANSWER: 2 Signs of digoxin toxicity include yellow vision and dysrhythmias. The furosemide diuretic increases urinary excretion of potassium and can cause hypokalemia. A low serum potassium level can contribute to both cardiac dysrhythmias and digoxin toxicity. The digoxin should be held until a serum digoxin level is determined. A serum potassium level of 2.9 mEq/L indicates hypokalemia, not hyperkalemia. The yellow vision is a characteristic sign of digoxin toxicity and not a sign of cerebral damage from an infarct or bleeding. ➧ Test-taking Tip: Carefully

A nurse is reviewing the chart of a client with a diagnosis of stage II heart failure (see abbreviated chart). The data should suggest to the nurse that: Admitting History & Physical Serum Laboratory Data Diagnostic Data Results Medications • Experiencing dyspnea on • BNP: 886 ng/L* exertion • aPTT: 55 sec** • Lung sounds: • K+: 2.9 mEq/L crackles in bilateral bases • Reports seeing yellow halo when visualizing objects *B-natriuretic peptide (BNP) normal value 100 ng/L **Activated partial thromboplastin time (aPTT) normal range 24 to 33 seconds • 12-lead ECG results: Atrial fibrillation with frequent PVCs. • Chest x-ray: Pulmonary infiltrated bilateral bases. Cardiomegaly • Buffered aspirin 325 mg oral daily • Digoxin (Lanoxin®) 0.25 mg oral daily • Furosemide (Lasix®) 40 mg oral daily • Atenolol (Tenormin®) 25 mg oral • Heparin intravenous infusion per protocol 1. medications should be administered as ordered. 2. the client may be experiencing digoxin toxicity. 3. hyperkalemia likely caused the client's cardiac dysrhythmias. 4. the client's visual disturbance likely resulted from the atrial fibrillation rhythm or effects of the anticoagulants. .

ANSWER: 2 Signs of digoxin toxicity include yellow vision and dysrhythmias. The furosemide diuretic increases urinary excretion of potassium and can cause hypokalemia. A low serum potassium level can contribute to both cardiac dysrhythmias and digoxin toxicity. The digoxin should be held until a serum digoxin level is determined. A serum potassium level of 2.9 mEq/L indicates hypokalemia, not hyperkalemia. The yellow vision is a characteristic sign of digoxin toxicity and not a sign of cerebral damage from an infarct or bleeding. ➧ Test-taking Tip: Carefully read the information in the client's chart and then read each of the options. Use the key words in the options to analyze the information provided in the client's chart, which are, respectively, "medications...appropriate," "digoxin toxicity," "hyperkalemia," and "visual disturbances." Then use the process of elimination

A 60-year-old client, admitted to a hospital with chest pain, has been functioning independently at home. During the night, the client is found wandering in the hallway and states, "I can't find my kitchen. I need a glass of milk." The nurse's best interpretation of the client's behavior is: 1. the client likely had a stroke. 2. the stress of being in unfamiliar surroundings has caused the client's confusion. 3. the decline in mental status, especially at night, is a normal part of aging. 4. this is an insidious change and it likely means the client has early dementia.

ANSWER: 2 Stress of unfamiliar situations or surroundings can cause confusion in an older adult. The client would be exhibiting other signs and symptoms if a stroke had occurred. Short-term memory is often less efficient, but a decline in mental status is not a normal part of aging. The confusion was an abrupt change in behavior, not insidious.

A client has a wound suction device for blood salvage following a left total knee replacement so that the blood can be reinfused into the client within the first 6 hours postoperatively. Which intervention should a nurse plan to implement to care for this wound suction device? 1. Discard the first 500 mL in the suction container and wait until the container is full again before beginning a reinfusion. 2. As soon as the prescribed amount is noted in the container, obtain the blood and prepare it for reinfusion into the client intravenously. 3. Separate the blood from the drainage, and reinfuse the blood back through the drainage system into the wound. 4. Remove the blood from the drainage system and send it to the blood bank to be prepared for an infusion.

ANSWER: 2 The blood salvage wound suction device drains blood from the surgical wound for reinfusion intravenously within 6 hours. The special device collects and filters the blood so it can be drained into a transfusion bag for infusion. The first 500 mL would be mostly blood and should not be discarded but reinfused. Usually all but 50 mL of the drainage in the collection device would be drained for infusion. There is no mechanism to separate the blood from other drainage. The blood is not reinfused into the wound but intravenously. There is no need to send the drainage to the blood bank. It will delay infusion, increase the risk of contamination, and increase the risk of administering mismatched blood. ➧ Test-taking Tip: Note that option 2 is more complete than the other options.

A nurse is assisting a physician with a cardioversion procedure. The client is anxious and fears the pain associated with the countershock. Conscious sedation is given. Monitoring of the client is especially important in conscious sedation due to which of the following? 1. Risk of hypertensive response 2. Potential loss of airway 3. Allergic reaction to sedatives 4. Change in level of consciousness

ANSWER: 2 The client's ability to maintain a patent airway is central to the decision to use conscious sedation. Oversedation may result in the loss of the airway. A hypertensive response is not associated with conscious sedation. Although an allergic reaction may occur, the loss of airway is a more common complication. A change in level of consciousness is expected. ➧Test-taking Tip: Use the ABCs (airway, breathing, circulation) to answer this question. Remember that the airway is especially importanT

9. A hospitalized client, identified to be at risk for thromboembolic disease, has anti-embolism hose ordered. A nurse discusses the correct use of the stockings. Which direction should the nurse include in teaching this client? 1. If ambulating 10 times daily for 5 minutes at a time, wearing the hose is unnecessary. 2. The most appropriate time to apply the hose is before standing to get out of bed in the morning. 3. If the hose becomes painful to the skin underneath, notify the nurse and request pain medication. 4. Only cross the legs while wearing the antiembolism hose; otherwise keep the legs uncrossed.

ANSWER: 2 The most appropriate time to apply anti-embolism stockings is before the client arises from bed. This maximizes the compression effect, thus lessening venous distension and development of edema. Frequent ambulation is a positive intervention to prevent thromboembolic disease but should be used in addition to wearing the anti-embolism stockings. If the stockings cause skin discomfort to the client, the stockings and skin underneath must be assessed. The stockings may need to be removed and then reapplied without twisting or wrinkles in the stockings. Crossing the legs impedes circulation and should be avoided with or without the elastic stockings. ➧ Test-taking Tip: The issue of the question is the correct use of anti-embolism stockings. Evaluate each option regarding safe use. Eliminate option 1 since it contradicts use of the elastic stockings

A nurse is working with a certified nursing assistant (CNA) and a licensed practical nurse (LPN) in providing care to a group of clients. Which tasks should the nurse assign to the CNA and LPN? 1. CNA to perform simple dressing changes; LPN to assess and care for two noncomplex clients 2. CNA to empty and record urinary catheter bag drainage; LPN to administer oral and intramuscular medications 3. CNA to assist clients with hygiene; LPN to provide postmortem care and meet with a deceased client's family 4. CNA to take and document vital signs on all clients; LPN to complete the discharge paperwork to be reviewed with two clients

ANSWER: 2 The scope of practice for a CNA includes measuring and recording intake and output and for the LPN to administer oral and intramuscular medications. A CNA is able in some facilities to perform a simple dressing change, but if the registered nurse (RN) changes it the RN would be able to assess the incision. An LPN should not be assessing clients. A CNA is able to assist with hygiene, but meeting with the family of a deceased client should be completed by the RN and not the LPN. A CNA is able to take and document vital signs, but the RN should be completing discharge paperwork to be reviewed with the clients. The discharge paperwork often includes a review of the care plan and addressing unmet needs of the client. ➧ Test-taking Tip: Eliminate options that include aspects of the RN role that should not be delegated, including assessment, evaluation, and education.

A nurse is assessing a malnourished adolescent who has been consuming a vegan diet. For signs of which specific vitamin deficiency should the nurse assess this client? 1. Vitamin A 2. Vitamin B12 3. Vitamin C 4. Vitamin K

ANSWER: 2 Vegans abstain from eating animal products, which provide vitamin B12. Fruits and vegetables that are eaten by vegans contain vitamins A, C, and K, so these are less likely to be deficient. ➧ Test-taking Tip:Think about how vitamins A, C, and K are similar and vitamin B12 is different.

79. A child is found to be deficient in iron. To increase the child's absorption of iron, which vitamin should a nurse encourage the parents to supplement? 1. Vitamin A 2. Vitamin C 3. Vitamin D 4. Vitamin E

ANSWER: 2 Vitamin C (ascorbic acid) facilitates iron absorption by acting on hydrochloric acid to keep iron in the more absorbable ferrous form. Vitamin A is essential to night vision, the health of epithelial tissue, normal bone growth, and energy regulation. Vitamin D is essential for absorption and use of calcium for bone and tooth growth. Vitamin E is an antioxidant that stimulates the immune system. ➧ Test-taking Tip: If unsure, look for similarities and differences; vitamins A, D, E, and K are fat-soluble vitamins, whereas vitamin C is a water-soluble vitamin. An option that is different is likely to be the answer.

A child is found to be deficient in iron. To increase the child's absorption of iron, which vitamin should a nurse encourage the parents to supplement? 1. Vitamin A 2. Vitamin C 3. Vitamin D 4. Vitamin E

ANSWER: 2 Vitamin C (ascorbic acid) facilitates iron absorption by acting on hydrochloric acid to keep iron in the more absorbable ferrous form. Vitamin A is essential to night vision, the health of epithelial tissue, normal bone growth, and energy regulation. Vitamin D is essential for absorption and use of calcium for bone and tooth growth. Vitamin E is an antioxidant that stimulates the immune system. ➧ Test-taking Tip: If unsure, look for similarities and differences; vitamins A, D, E, and K are fat-soluble vitamins, whereas vitamin C is a water-soluble vitamin. An option that is different is likely to be the answer.

A nurse working on a telemetry unit is planning to complete noon assessments for four assigned clients with type 1 diabetes mellitus. All of the clients received subcutaneous insulin aspart (NovoLog®) at 0800 hours. In which order should the nurse assess the clients? Place each answer option into the correct order. _____ A 60-year-old client who is nauseous and has just vomited for the second time _____ A 45-year-old client who is dyspneic and has chest pressure and new onset atrial fibrillation _____ A 75-year-old client with a fingerstick blood glucose level of 300 mg/dL _____ A 50-year-old client with a fingerstick blood glucose level of 70 mg/dL

ANSWER: 2, 1, 3, 4 First, the nurse should assess the client with new onset atrial fibrillation and dyspnea. Diabetes increases the risk of coronary artery disease and myocardial infarction. Next, assess the client who just vomited. The client with a 300 mg/dL blood glucose level should then be assessed. This blood glucose level is not immediately life-threatening, but needs to be lowered as soon as possible. The client with the blood glucose level of 70 mg/dL can be assessed last because this is a normal blood glucose level. ➧ Test-taking Tip: Use the ABC's (airway, breathing, circulation) to establish the priority client. Then, look at the information provided for each client to determine the next priority. Consider that all clients received insulin at 0800 hours

54. A nurse is caring for a client hospitalized with idiopathic thrombocytopenic purpura (ITP). Which selfcare measures should the nurse plan to include when teaching the client? SELECT ALL THAT APPLY. 1. "Use dental floss after brushing your teeth to prevent gum hyperplasia." 2. "Use only an electric razor when shaving." 3. "Remove throw rugs in your home, and avoid clutter." 4. "Increase fiber in your diet, and drink plenty of liquids to avoid constipation." 5. "Keep appointments for monthly platelet transfusions."

ANSWER: 2, 3, 4 Because the client is at risk for bleeding due to low platelets counts, measures to decrease the risk of bleeding should be implemented. Throw rugs and clutter increase the risk for falls with subsequent bleeding. Constipation can lead to hemorrhoids and increase the risk for bleeding. Dental floss can traumatize the gums and increase the risk for bleeding. Platelet transfusions are usually avoided because the person's antiplatelet antibodies bind with the transfused platelets, causing them to be destroyed. ➧ Test-taking Tip: If unsure of ITP, recall that purpura is caused from bleeding into the tissues. Select the options that would prevent bleeding.

A nurse is caring for a client hospitalized with idiopathic thrombocytopenic purpura (ITP). Which selfcare measures should the nurse plan to include when teaching the client? SELECT ALL THAT APPLY. 1. "Use dental floss after brushing your teeth to prevent gum hyperplasia." 2. "Use only an electric razor when shaving." 3. "Remove throw rugs in your home, and avoid clutter." 4. "Increase fiber in your diet, and drink plenty of liquids to avoid constipation." 5. "Keep appointments for monthly platelet transfusions."

ANSWER: 2, 3, 4 Because the client is at risk for bleeding due to low platelets counts, measures to decrease the risk of bleeding should be implemented. Throw rugs and clutter increase the risk for falls with subsequent bleeding. Constipation can lead to hemorrhoids and increase the risk for bleeding. Dental floss can traumatize the gums and increase the risk for bleeding. Platelet transfusions are usually avoided because the person's antiplatelet antibodies bind with the transfused platelets, causing them to be destroyed. ➧ Test-taking Tip: If unsure of ITP, recall that purpura is caused from bleeding into the tissues. Select the options that would prevent bleeding.

Which nursing actions should a nurse plan when caring for a client experiencing dyspnea due to heart failure and chronic obstructive pulmonary disease (COPD)? SELECT ALL THAT APPLY. 1. Apply oxygen 6 liters per nasal cannula 2. Elevate the head of the bed 30 to 40 degrees 3.Weigh client daily in the morning 4. Teach client pursed-lip breathing techniques 5. Turn and reposition the client every 1 to 2 hours

ANSWER: 2, 3, 4 Elevating the head of the bed will promote lung expansion. Daily weights will assess fluid retention. Fluid volume excess can increase dyspnea and cause pulmonary edema. Pursed-lip breathing techniques allow the client to conserve energy and slow the breathing rate. Applying greater than 4 liters of oxygen per nasal cannula is contraindicated for COPD. High flow rates can depress the hypoxic drive. Because the client with COPD suffers from chronically high CO2 levels, the stimulus to breathe is the low O2 level (a hypoxic drive). The situation does not warrant turning the client every 1 to 2 hours. This activity could increase the client's energy expenditure and dyspnea

Which nursing actions should a nurse plan when caring for a client experiencing dyspnea due to heart failure and chronic obstructive pulmonary disease (COPD)? SELECT ALL THAT APPLY. 1. Apply oxygen 6 liters per nasal cannula 2. Elevate the head of the bed 30 to 40 degrees 3.Weigh client daily in the morning 4. Teach client pursed-lip breathing techniques 5. Turn and reposition the client every 1 to 2 hours

ANSWER: 2, 3, 4 Elevating the head of the bed will promote lung expansion. Daily weights will assess fluid retention. Fluid volume excess can increase dyspnea and cause pulmonary edema. Pursed-lip breathing techniques allow the client to conserve energy and slow the breathing rate. Applying greater than 4 liters of oxygen per nasal cannula is contraindicated for COPD. High flow rates can depress the hypoxic drive. Because the client with COPD suffers from chronically high CO2 levels, the stimulus to breathe is the low O2 level (a hypoxic drive). The situation does not warrant turning the client every 1 to 2 hours. This activity could increase the client's energy expenditure and dyspnea. ➧ Test-taking Tip: Focus on the key words "dyspnea," "heart failure," and "COPD." Recall that dyspnea from heart failure is caused by fluid volume overload whereas dyspnea from COPD is caused from airway noncompliance. Combine actions for both of these problems in selecting the correct options.

61. At 1000 hours, a nurse is documenting after administering 275 mL of compatible platelets, unit number XR123, to a client. Which information should the nurse document? SELECT ALL THAT APPLY. 1. "One unit blood infused over 4 hours." 2. "Platelet number XR123 checked prior to administration." 3. "No transfusion reactions noted." 4. "D5W infused with platelets to prevent cell clumping." 5. "Infusion of 275 mL of platelets started at 0830 hours completed."

ANSWER: 2, 3, 5 Documentation should include the type of product infused (platelets), product number (compatible platelets were ordered), volume infused, time of infusion, and any adverse reactions. Platelets should be infused as fast as the client can tolerate the infusion to diminish clumping. Option 1 documents an incomplete blood type, and platelets are unlikely to be administered over 4 hours. Only 0.9% NaCl should be used when administering blood or blood products and usually only to purge the line before and after administration.

At 1000 hours, a nurse is documenting after administering 275 mL of compatible platelets, unit number XR123, to a client. Which information should the nurse document? SELECT ALL THAT APPLY. 1. "One unit blood infused over 4 hours." 2. "Platelet number XR123 checked prior to administration." 3. "No transfusion reactions noted." 4. "D5W infused with platelets to prevent cell clumping." 5. "Infusion of 275 mL of platelets started at 0830 hours completed."

ANSWER: 2, 3, 5 Documentation should include the type of product infused (platelets), product number (compatible platelets were ordered), volume infused, time of infusion, and any adverse reactions. Platelets should be infused as fast as the client can tolerate the infusion to diminish clumping. Option 1 documents an incomplete blood type, and platelets are unlikely to be administered over 4 hours. Only 0.9% NaCl should be used when administering blood or blood products and usually only to purge the line before and after administration. ➧ Test-taking Tip: Apply knowledge of information that should be documented in the client's chart when administering blood or blood products.

A nurse is teaching a client newly diagnosed with chronic stable angina. Which instructions should the nurse incorporate in the teaching session on measures to prevent future angina? SELECT ALL THAT APPLY. 1. Increase isometric arm exercises to build endurance. 2.Wear a face mask when outdoors in cold weather. 3. Take nitroglycerin before a stressful situation even though pain is not present. 4. Perform most exertional activities in the morning. 5. Avoid straining at stool. 6. Eliminate tobacco use.

ANSWER: 2, 3, 5, 6 Blood vessels constrict in response to cold and increase the workload of the heart. Sexual activity and straining at stool increases sympathetic stimulation and cardiac workload. Nitroglycerin produces vasodilation and improves blood flow to the coronary arteries; it can be used prophylactically to prevent angina. Nicotine stimulates catecholamine release, producing vasoconstriction and an increased heart rate. Isometric exercise of the arms can cause exertional angina. Exertional activity increases the heart rate, thus reducing the time the heart is in diastole, when blood flow to the coronary arteries is the greatest. A period of rest should occur between activities and activities should be spaced. ➧ Test-taking Tip:The key phrase is "measures to prevent future angina." Consider if each option could potentially increase myocardial oxygen demand or decrease available oxygen, either of which could precipitate angina.

A nurse is teaching a client newly diagnosed with chronic stable angina. Which instructions should the nurse incorporate in the teaching session on measures to prevent future angina? SELECT ALL THAT APPLY. 1. Increase isometric arm exercises to build endurance. 2.Wear a face mask when outdoors in cold weather. 3. Take nitroglycerin before a stressful situation even though pain is not present. 4. Perform most exertional activities in the morning. 5. Avoid straining at stool. 6. Eliminate tobacco use.

ANSWER: 2, 3, 5, 6 Blood vessels constrict in response to cold and increase the workload of the heart. Sexual activity and straining at stool increases sympathetic stimulation and cardiac workload. Nitroglycerin produces vasodilation and improves blood flow to the coronary arteries; it can be used prophylactically to prevent angina. Nicotine stimulates catecholamine release, producing vasoconstriction and an increased heart rate. Isometric exercise of the arms can cause exertional angina. Exertional activity increases the heart rate, thus reducing the time the heart is in diastole, when blood flow to the coronary arteries is the greatest. A period of rest should occur between activities and activities should be spaced. ➧ Test-taking Tip:The key phrase is "measures to prevent future angina." Consider if each option could potentially increase myocardial oxygen demand or decrease available oxygen, either of which could precipitate angina.

Which points should the nurse plan to include when teaching a client receiving a thiazide diuretic? SELECT ALL THAT APPLY. 1. Take the radial pulse before setting up the medication. 2. Include fruits such as melons and bananas in the diet. 3. Report side effects such as muscle cramps, nausea, or a skin rash. 4. Self-administer the last dose at bedtime when fluid tends to be at the highest levels. 5. Keep appointments for laboratory monitoring, including serum electrolytes, glucose, creatinine, blood urea nitrogen (BUN), and uric acid levels. 6. Minimize intake of high-fat foods because thiazide diuretics can increase serum cholesterol, low-density lipoprotein (LDL), and triglyceride levels.

ANSWER: 2, 3, 5, 6 Thiazide diuretics can cause hypokalemia. Encouraging potassiumrich foods can help maintain potassium levels. Muscle cramps are a sign of possible medication side effects of hypokalemia and hypocalcemia. Nausea and rash are also medication side effects. Laboratory results to be monitored during thiazide diuretic therapy include serum electrolytes, glucose, creatinine, BUN, and uric acid levels because thiazide diuretics are excreted mainly unchanged by the kidneys and can increase glucose levels. Reminders regarding keeping appointments promote medication adherence. Thiazide diuretics can increase serum cholesterol, LDL, and triglyceride levels, so teaching the client to minimize high-fat foods will help maintain cholesterol levels. It is unnecessary for a client to monitor the pulse before taking thiazide diuretics. A diuretic taken at bedtime can cause nocturia and loss of sleep. The usual timing of the last daily dose of a diuretic is at suppertime. ➧ Test-taking Tip: Recall that thiazide diuretics lower blood pressure through diuresis, which can result in hypokalemia. Focus on the common thiazide diuretic, hydrochlorothiazide, and its effects and side effects when reading each of the options.

Because a step-down cardiac unit is unusually busy, a nurse fails to obtain vital signs at 0200 hours for a client 2 days postoperative for a mitral valve replacement. The client was stable when assessed at 0600 hours, so the nurse documents the electrocardiogram monitor's heart rate in the client's medical record for both the 0400 and 0600 vital signs. The charge nurse supervising the nurse determines that the nurse's behavior was: SELECT ALL THAT APPLY. 1. the correct action because neither complications nor harmful effects occurred. 2. a legal issue because the nurse has fraudulently falsified documentation. 3. demonstrating beneficence because the nurse decided what was best for the client. 4. an ethical issue of veracity because the nurse has been untruthful regarding the client's care. 5. an ethical legal issue of confidentiality because the nurse disclosed incorrect information. 6. demonstrating distributive justice because the nurse decided other clients' needs were priority.

ANSWER: 2, 4 Documenting vital signs that the nurse did not obtain is both a legal and ethical concern because documents were falsified and the nurse was untruthful regarding obtaining the vital signs. Veracity is telling the truth and not lying or deceiving others. Even if harm had not occurred, the nurse's behavior of falsifying documentation poses an ethicallegal concern and is never the correct action. Beneficence is doing good. There is no information to indicate the nurse did what was best for the client. Confidentiality relates to privacy and not disclosing private information about another. Documenting incorrect vital signs is not disclosing confidential information. Distributive justice is the distribution of resources to clients. There is no information about the resources available to the nurse. ➧ Test-taking Tip: Focus on the nurse's behavior of falsifying documentation. Avoid reading into the question. Despite the unit being unusually busy, there is no information as to what the nurse was doing during the shift. Eliminate the options that are suggestive of nurse actions other than the behaviors presented.

. A nurse is working with a certified nursing assistant (CNA) providing care for four clients on a busy telemetry unit. All four clients are in need of immediate attention. The CNA is a senior nursing student who has been administering medications and performing procedures during clinical experiences as a student nurse. The charge nurse supervising care on the telemetry unit determines that care is appropriate when the registered nurse (RN) working with the CNA delegates: SELECT ALL THAT APPLY. 1. administering acetaminophen (Tylenol®) to the client with an elevated temperature. 2. taking vital signs on the client newly admitted with a diagnosis of heart failure. 3. finishing the discharge instructions so the client with a new pacemaker implant can go home. 4. changing a client's chest tube dressing because it got wet when the water pitcher overturned. 5. providing a sponge bath for the client with the elevated temperature. 6. checking the lung sounds of the client whose chest tube drainage system was tipped over and then righted

ANSWER: 2, 5

A nurse is working with a certified nursing assistant (CNA) providing care for four clients on a busy telemetry unit. All four clients are in need of immediate attention. The CNA is a senior nursing student who has been administering medications and performing procedures during clinical experiences as a student nurse. The charge nurse supervising care on the telemetry unit determines that care is appropriate when the registered nurse (RN) working with the CNA delegates: SELECT ALL THAT APPLY. 1. administering acetaminophen (Tylenol®) to the client with an elevated temperature. 2. taking vital signs on the client newly admitted with a diagnosis of heart failure. 3. finishing the discharge instructions so the client with a new pacemaker implant can go home. 4. changing a client's chest tube dressing because it got wet when the water pitcher overturned. 5. providing a sponge bath for the client with the elevated temperature. 6. checking the lung sounds of the client whose chest tube drainage system was tipped over and then righted

ANSWER: 2, 5 Legally a student nurse employed as a nursing assistant in a facility is only allowed to perform tasks listed in the job description of a nursing assistant even though the student nurse has received instruction and acquired competence in administering medications and performing sterile procedures. The tasks of a nursing assistant include taking vital signs and bathing clients. Medication administration, teaching, sterile procedures, and assessments are not within the nursing assistant's scope of practice. ➧ Test-taking Tip: Read the information given in the question carefully. The issue of the question is tasks that the RN can legally delegate to a CNA who is also a student nurse. Delegated tasks must be within the job description of the nursing assistant

49. A nurse is working with a certified nursing assistant (CNA) providing care for four clients on a busy telemetry unit. All four clients are in need of immediate attention. The CNA is a senior nursing student who has been administering medications and performing procedures during clinical experiences as a student nurse. The charge nurse supervising care on the telemetry unit determines that care is appropriate when the registered nurse (RN) working with the CNA delegates: SELECT ALL THAT APPLY. 1. administering acetaminophen (Tylenol®) to the client with an elevated temperature. 2. taking vital signs on the client newly admitted with a diagnosis of heart failure. 3. finishing the discharge instructions so the client with a new pacemaker implant can go home. 4. changing a client's chest tube dressing because it got wet when the water pitcher overturned. 5. providing a sponge bath for the client with the elevated temperature. 6. checking the lung sounds of the client whose chest tube drainage system was tipped over and then righted

ANSWER: 2, 5 Legally a student nurse employed as a nursing assistant in a facility is only allowed to perform tasks listed in the job description of a nursing assistant even though the student nurse has received instruction and acquired competence in administering medications and performing sterile procedures. The tasks of a nursing assistant include taking vital signs and bathing clients. Medication administration, teaching, sterile procedures, and assessments are not within the nursing assistant's scope of practice. ➧ Test-taking Tip: Read the information given in the question carefully. The issue of the question is tasks that the RN can legally delegate to a CNA who is also a student nurse. Delegated tasks must be within the job description of the nursing assistant.

89. A nurse is reviewing a serum laboratory report for a client following surgery. Results include WBCs, 18,000 K/μL; SCr, 2.2 mg/dL; K, 3.5 mEq/L; and Hgb, 6.8 mg/dL. Per the physician's orders, the nurse assesses the client, removes the subclavian venous access device, and sends the tip for culture. In which order should the nurse perform the remaining physician's orders? Prioritize the nursing actions. _____ Administer cephazolin sodium (Ancef®) 1 gram intravenously (IV) _____ Administer 1 unit packed red blood cells _____ Prime the blood tubing with 0.9% NaCl _____ Insert a new intravenous access device _____ Send the nursing assistant to obtain the blood from blood bank _____ Verify the client's identification and complete the checks for safe administration of the blood product

ANSWER: 2, 6, 4, 1, 3, 5 First insert an intravenous access device and then start the Ancef®. It will take about 20 minutes to infuse the Ancef®. Because blood would need to be returned to the blood bank if out for more than 30 minutes, the IV should be started before obtaining the blood. Next, send the nursing assistant to retrieve the blood from the blood bank. Prime the blood tubing with 0.9% NaCl infusion while waiting for the blood (the blood tubing would not be used for administering Ancef®). Complete the identification and blood administration checks to ensure it is the correct blood product for this client, and then start the blood infusion. The blood should infuse between 2 and 4 hours.

A client diagnosed with class II heart failure according to the New York Heart Association Functional Classification has been taught about the initial treatment plan for this disease. A nurse determines that the client needs additional teaching if the client states that the treatment plan includes: 1. diuretics. 2. a low-sodium diet. 3. home oxygen therapy. 4. angiotensin-converting enzyme (ACE) inhibitors.

ANSWER: 3

A registered nurse (RN) assesses that a client is pale, diaphoretic, dyspneic, and experiencing chest pain. Which actions are best for the nurse to take? 1. Stay with the client, call the charge nurse for help, and call the patient care assistant (PCA) to bring an automatic vital signs machine to the room immediately. 2. Call the PCA to take the client's vital signs while the RN leaves to obtain a narcotic analgesic for administration and notify the charge nurse. 3. Apply oxygen, call the PCA to bring an automatic vital signs machine, and call the charge nurse for help and ask to bring the chart and morphine sulfate noted on the medication record. 4. Activate the emergency system for a code to get immediate help, apply oxygen, and send responders for needed equipment and medication.

ANSWER: 3

After an inferior-septal wall myocardial infarction, which complication should a nurse suspect when noting jugular venous distention (JVD) and ascites? 1. Left-sided heart failure 2. Pulmonic valve malfunction 3. Right-sided heart failure 4. Ruptured septum

ANSWER: 3

An 87-year-old client has severe coronary artery disease and has been advised to complete a living will and a durable power of attorney for health care. The client asks, "Why do I need both?" A nurse explains that a living will differs from a durable power of attorney in that a living will: 1. is an example of an advanced health-care directive. 2. allows the client to designate a person to make decisions should the client become unable to provide informed consent for health-care decisions. 3. provides a legal document for the client to specify what type of medical treatment is desired should the client becomes incapacitated and terminally ill. 4. is not a legal document, but makes it easier and quicker for medical personnel to care for the client if the client becomes terminally ill.

ANSWER: 3 A living will is a legal document allowing the person to specify what type of medical treatment is desired if the person becomes incapacitated or terminally ill. Both the living will and the health-care power of attorney are advance health-care directives and legal documents.

55. A nurse teaches a coworker that the treatment for hemophilia will likely include periodic selfadministration of: 1. platelets. 2. whole blood. 3. factor concentrates. 4. fresh frozen plasma.

ANSWER: 3 A person with hemophilia A is deficient in factor VIII; hemophilia B, factor IX; and von Willebrand's hemophilia, the von Willebrand's factor and factor VIII. Recombinant forms of the factors are available for the client to self-administer intravenously at home. Although whole blood and fresh frozen plasma contain the deficient factors, periodic administration of factor concentrates are safer. Platelets do not contain the deficient clotting factors.

A nurse teaches a coworker that the treatment for hemophilia will likely include periodic selfadministration of: 1. platelets. 2. whole blood. 3. factor concentrates. 4. fresh frozen plasma.

ANSWER: 3 A person with hemophilia A is deficient in factor VIII; hemophilia B, factor IX; and von Willebrand's hemophilia, the von Willebrand's factor and factor VIII. Recombinant forms of the factors are available for the client to self-administer intravenously at home. Although whole blood and fresh frozen plasma contain the deficient factors, periodic administration of factor concentrates are safer. Platelets do not contain the deficient clotting factors. ➧ Test-taking Tip: Apply knowledge of blood products and factor deficiencies in hemophilia.

33. A client is hospitalized for heart failure secondary to alcohol-induced cardiomyopathy. The client is started on milrinone (Primacor®) and placed on a transplant waiting list. The client has been curt and verbally aggressive in expressing dissatisfaction with the medication orders, overall care, and the need for energy conservation. A nurse should interpret that the client's behavior is likely related to the client's: 1. denial of the illness. 2. reaction to milrinone (Primacor®). 3. fear of the diagnosis. 4. response to cerebral anoxia.

ANSWER: 3 A threatening situation (need for heart transplant) can produce fear. Fear and helplessness may cause a client to verbally attack health team members to maintain control. There is no supporting evidence that the client denies the existence of a health problem. Minimizing symptoms or noncompliant behaviors would indicate denial. Milrinone is used in short-term treatment of congestive heart failure unresponsive to conventional therapy with digoxin, diuretics, and vasodilators. It increases myocardial contractility and decreases preload and afterload by direct dilating effect on the vascular smooth muscle. It does not cause behavior changes. Although a low cardiac output may lead to cerebral anoxia, there is insufficient evidence in this situation to support the conclusion of cerebral anoxia causing the client's reaction. ➧ Test-taking Tip: Focus on the issue, the client's reaction to the illness and hospitalization.

A nurse is assessing a client diagnosed with an anterior-lateral myocardial infarction (MI). The nurse adds a nursing diagnosis to the client's plan of care of decreased cardiac output when which finding is noted on assessment? 1. One-sided weakness 2. Presence of an S4 heart sound 3. Crackles auscultated in bilateral lung bases 4. Vesicular breath sounds over lung lobes

ANSWER: 3 An anterior-lateral MI can produce left ventricular dysfunction and low cardiac output. With low cardiac output, blood accumulates in the heart and backs up into the pulmonary system. The increased pulmonary pressure causes fluid to move into interstitial spaces and then into the alveoli. One-sided weakness suggests complications of a cerebrovascular accident, which can be caused from a clot or plaque embolus secondary to the MI. An S4 heart sound is produced when blood flows forcefully from the atrium to a resistant ventricle during late ventricular diastole. Vesicular breath sounds are normal over lesser bronchi, bronchioles, and lobes of the lung. ➧ Test-taking Tip: Recall that an anterior MI can produce left ventricular dysfunction. Focus on the issue of an assessment finding indicative of low cardiac output. Eliminate option 1 because it relates to tissue perfusion. If unsure of option 2, move to option 3. Note that options 3 and 4 relate to the lungs, so one or both of these must be wrong. Recall that crackles are always abnormal, whereas the presence of S4 can be normal in an older adult, and vesicular lung sounds located over lung lobes is a normal finding.

A nurse is assessing a client diagnosed with an anterior-lateral myocardial infarction (MI). The nurse adds a nursing diagnosis to the client's plan of care of decreased cardiac output when which finding is noted on assessment? 1. One-sided weakness 2. Presence of an S4 heart sound 3. Crackles auscultated in bilateral lung bases 4. Vesicular breath sounds over lung lobes

ANSWER: 3 An anterior-lateral MI can produce left ventricular dysfunction and low cardiac output. With low cardiac output, blood accumulates in the heart and backs up into the pulmonary system. The increased pulmonary pressure causes fluid to move into interstitial spaces and then into the alveoli. One-sided weakness suggests complications of a cerebrovascular accident, which can be caused from a clot or plaque embolus secondary to the MI. An S4 heart sound is produced when blood flows forcefully from the atrium to a resistant ventricle during late ventricular diastole. Vesicular breath sounds are normal over lesser bronchi, bronchioles, and lobes of the lung. ➧ Test-taking Tip: Recall that an anterior MI can produce left ventricular dysfunction. Focus on the issue of an assessment finding indicative of low cardiac output. Eliminate option 1 because it relates to tissue perfusion. If unsure of option 2, move to option 3. Note that options 3 and 4 relate to the lungs, so one or both of these must be wrong. Recall that crackles are always abnormal, whereas the presence of S4 can be normal in an older adult, and vesicular lung sounds located over lung lobes is a normal finding.

A client, following a total hip replacement, asks a nurse why she is receiving enoxaparin (Lovenox®) for prevention of deep vein thrombosis (DVT) when, with her last hip surgery, she received heparin subcutaneously. What is the nurse's best response? 1. "Enoxaparin is less expensive and easier to administer than the heparin." 2. "There is less risk of bleeding with enoxaparin, and it doesn't affect your laboratory results." 3. "Enoxaparin is a low-molecular-weight heparin that lasts twice as long as regular heparin." 4. "Enoxaparin can be administered orally whereas heparin is only administered by injection."

ANSWER: 3 Because enoxaparin is more specific in inhibiting active factor X, the response is more stable and the effect is two to four times longer than that of heparin. The cost of enoxaparin is more than twice the cost of the equivalent dose of heparin per injection. Both enoxaparin and heparin are administered by subcutaneous injection, and both are available in prefilled syringes for injection. Both enoxaparin and heparin increase activated partial thromboplastin time (aPTT). Enoxaparin is only administered subcutaneously. ➧ Test-taking Tip: Focus on the issue: the difference between enoxaparin and heparin. Note that options 1, 2, and 4 are similar, addressing the supposed benefits of enoxaparin, whereas option 3 is different, describing the action of enoxaparin. The option that is different is usually the answer.

A nurse is caring for a client who has 0.9% NaCl infusing intravenously (IV). An order had been written the previous day to change the IV solution to 0.9% NaCl with 10 mEq KCL. Which action should the nurse initiate first? 1. Notify the client's physician 2. Complete an incident report 3. Check the client's serum potassium level 4. Replace 0.9% NaCl with the ordered solution

ANSWER: 3 Because the order was written the previous day and not implemented, the nurse should first check the client's serum potassium level and then notify the physician. The nurse should determine if there are order changes before replacing the solution. The physician may change the amount of KCL in the IV solution based on the client's serum potassium level and other client data. An incident report should be completed by the nurse after caring for the client. ➧ Test-taking Tip: Note the key word "first." Remember that KCL is potassium chloride.

A nurse should inform a nursing assistant to avoid taking a rectal temperature for which client? 1. The adult client who underwent ileostomy surgery because of a perforated bowel 2. The adult client who has a frequent, productive cough and is receiving oxygen by nasal cannula 3. The adult client who developed thrombocytopenia after receiving chemotherapy 4. The adult client with hypothermia

ANSWER: 3 Clients with thrombocytopenia have lower than normal levels of platelets and are at increased risk of bleeding. Measuring the temperature rectally exposes the client to the risk of rectal bleeding. There are no contraindications for using the rectal method to assess temperature in the other clients. In fact, monitoring rectal temperature is often used in clients with hypothermia since the temperature is too low to be measured orally. ➧ Test-taking Tip: The question calls for a negative answer. Evaluate each option and consider whether taking a temperature rectally would harm the clienT

95. A nurse completes the measurements for the electrocardiogram (ECG) monitor rhythm strips for four different clients. Following analysis of the ECG rhythms, the nurse determines that one of the clients requires an immediate intervention. Which measurement findings warrant immediate intervention? 1. PRI = 0.20 sec.; QRS = 0.10 sec.; QT = 0.38 sec.; ventricular rate = 102 beats per minute (bpm) 2. P waves fibrillatory; PR unmeasureable; QRS = .08 sec.; QT unmeasureable; ventricular rate = 90 bpm 3. PRI inconsistent; more Ps than QRSs and no correlation between Ps and QRSs; QRS = .14 sec; atrial rate = 60 bpm; ventricular rate = 30 bpm 4. PRI = 0.24 sec.; QRS = .10 sec.; QT = .44 sec.; ventricular rate = 58 bpm

ANSWER: 3 Complete heart block is a life-threatening dysrhythmia because, if untreated, it results in reduced cardiac output and subsequent ischemia. The atrial rate is usually sinus, 60 to 100 bpm. If the AV node initiates an impulse, the ventricular rate is 40 to 60 bpm; if the His Purkinje system initiates an impulse, it is 20 to 40 bpm. The QRS is widened if the impulse is initiated below the bundle of His. The PRI is variable and there is no relationship between the P waves and the QRS complexes

80. An increase in which specific serum laboratory test should indicate to a nurse that a high-iron diet for a client with early-stage iron-deficiency anemia has been effective? 1. Hemoglobin 2. Folate 3. Ferritin 4. Vitamin B12 ANSWER: 3 Ferritin levels reflect the available iron stores in the body and are specific to iron-deficiency anemia. A level less than 10 ng/mL is diagnostic of iron-deficiency anemia. As the condition improves, ferritin levels rise. In iron deficiency, the body cannot synthesize hemoglobin, but hemoglobin levels drop fairly late in the development of iron-deficiency anemia. Also, other nutrient deficiencies and medical conditions can affect hemoglobin levels. Serum folate is specific to folate-deficiency anemia.

ANSWER: 3 Ferritin levels reflect the available iron stores in the body and are specific to iron-deficiency anemia. A level less than 10 ng/mL is diagnostic of iron-deficiency anemia. As the condition improves, ferritin levels rise. In iron deficiency, the body cannot synthesize hemoglobin, but hemoglobin levels drop fairly late in the development of iron-deficiency anemia. Also, other nutrient deficiencies and medical conditions can affect hemoglobin levels. Serum folate is specific to folate-deficiency anemia.

An increase in which specific serum laboratory test should indicate to a nurse that a high-iron diet for a client with early-stage iron-deficiency anemia has been effective? 1. Hemoglobin 2. Folate 3. Ferritin 4. Vitamin B12

ANSWER: 3 Ferritin levels reflect the available iron stores in the body and are specific to iron-deficiency anemia. A level less than 10 ng/mL is diagnostic of iron-deficiency anemia. As the condition improves, ferritin levels rise. In iron deficiency, the body cannot synthesize hemoglobin, but hemoglobin levels drop fairly late in the development of iron-deficiency anemia. Also, other nutrient deficiencies and medical conditions can affect hemoglobin levels. Serum folate is specific to folate-deficiency anemia. ➧ Test-taking Tip: Note the key words "specific" and "irondeficiency." Use knowledge of terminology to answer this question (ferritin is an iron-phosphorus-protein complex).

81. A nurse teaches a client who is experiencing irondeficiency anemia to eat foods high in iron and foods that contain vitamin C at the same meal to increase the iron absorption. The nurse evaluates that teaching is effective when which meal is selected by the client? 1. Yogurt and oranges 2. Shrimp and potatoes 3. Lean beef steak and broccoli 4. Chicken and leafy green vegetables

ANSWER: 3 Good sources of iron include lean beef steak, steamed clams, navy beans, enriched cereal, cooked spinach, cooked Swiss chard, beef liver, and black beans. Three ounces of steamed clams have 23.8 mg of iron, compared with cooked beef liver, which has 5.24 mg. Dark green vegetables such as broccoli and bell peppers, citrus fruits, cabbage-type vegetables, cantaloupe, strawberries, lettuce, tomatoes, potatoes, papayas, and mangos are significant sources of vitamin C. Yogurt, shrimp, and chicken contain less iron than do steamed clams.

78. A nurse teaches a client who is experiencing irondeficiency anemia to eat foods high in iron and foods that contain vitamin C at the same meal to increase the iron absorption. The nurse evaluates that teaching is effective when which meal is selected by the client? 1. Yogurt and oranges 2. Shrimp and potatoes 3. Lean beef steak and broccoli 4. Chicken and leafy green vegetables

ANSWER: 3 Good sources of iron include lean beef steak, steamed clams, navy beans, enriched cereal, cooked spinach, cooked Swiss chard, beef liver, and black beans. Three ounces of steamed clams have 23.8 mg of iron, compared with cooked beef liver, which has 5.24 mg. Dark green vegetables such as broccoli and bell peppers, citrus fruits, cabbage-type vegetables, cantaloupe, strawberries, lettuce, tomatoes, potatoes, papayas, and mangos are significant sources of vitamin C. Yogurt, shrimp, and chicken contain less iron than do steamed clams. ➧ Test-taking Tip: Use knowledge of foods high in vitamin C and iron to answer this question.

A nurse teaches a client who is experiencing irondeficiency anemia to eat foods high in iron and foods that contain vitamin C at the same meal to increase the iron absorption. The nurse evaluates that teaching is effective when which meal is selected by the client? 1. Yogurt and oranges 2. Shrimp and potatoes 3. Lean beef steak and broccoli 4. Chicken and leafy green vegetables

ANSWER: 3 Good sources of iron include lean beef steak, steamed clams, navy beans, enriched cereal, cooked spinach, cooked Swiss chard, beef liver, and black beans. Three ounces of steamed clams have 23.8 mg of iron, compared with cooked beef liver, which has 5.24 mg. Dark green vegetables such as broccoli and bell peppers, citrus fruits, cabbage-type vegetables, cantaloupe, strawberries, lettuce, tomatoes, potatoes, papayas, and mangos are significant sources of vitamin C. Yogurt, shrimp, and chicken contain less iron than do steamed clams. ➧ Test-taking Tip: Use knowledge of foods high in vitamin C and iron to answer this question.

home health nurse visiting an older adult client for the first time is reviewing the client's medication list prepared by the client's daughter (see exhibit). Which finding in the client's medication list should be of most concern to the nurse and should be discussed first with the client? Aspart insulin 1 unit per 1 carbohydrate choice (CHO) taken at meals Aspirin 1 tab. every AM Atenolol (Tenormin®) 25 mg oral at breakfast Hydrochlorothiazide + captopril (Capozide®) 1 daily Captopril (Capoten®) 25 mg three times daily Glyberide (Diabeta®) 1 at breakfast Hydrochlorothiazide 25 mg twice daily Simvastatin (Zocor®) 10 mg daily in the evening 1. Some doses of medication are missing. 2. Some routes of medication are missing. 3. Some medications are duplicated. 4. Some medications have drug-drug interactions.

ANSWER: 3 Hydrochlorothiazide + captopril (Capozide®) is a combination product of hydrochlorothiazide and captopril. The nurse should first determine if the client is taking the combination product along with the individual products due to the potential for overdosing. Duplication of medications needs to be identified before missing dosages, missing routes, or drug-drug interactions are determined. The client may be clear regarding the dose and the route, but may not realize that two medications were replaced with one combination product. ➧ Test-taking Tip:The key word is "first." The greatest safety risk for the client is medication overdosing.

92. A nurse is caring for a client who is to receive a unit of packed red blood cells (PRBCs). Which safety measure should the nurse implement when administering the blood transfusion? 1. Monitor vital signs every 15 minutes during the transfusion to detect complications early 2. Administer the unit of PRBCs slowly over 5 hours to prevent a transfusion reaction 3. Stop the transfusion if a reaction occurs and administer 0.9% NaCl at the IV catheter hub to keep the intravenous (IV) site patent 4. Deliver the PRBCs through an infusion pump with standard tubing to ensure a consistent rate

ANSWER: 3 If the client experiences symptoms of a transfusion reaction, the transfusion should be stopped immediately. The line should be kept open for emergency medications, but saline should not be administered through the existing blood tubing because the client will receive the blood remaining in the tubing. Vital signs are assessed before the transfusion begins, 15 minutes after it is started, and then hourly until the transfusion is completed. Upon completion, the vital signs are taken and then repeated an hour later. PRBCs should infuse in 4 hours or less to avoid the risk of septicemia. Special tubing with a filter is used to administer blood.

according to the New York Heart Association Functional Classification has been taught about the initial treatment plan for this disease. A nurse determines that the client needs additional teaching if the client states that the treatment plan includes: 1. diuretics. 2. a low-sodium diet. 3. home oxygen therapy. 4. angiotensin-converting enzyme (ACE) inhibitors.

ANSWER: 3 In class II heart failure, normal physical activity results in fatigue, dyspnea, palpitations, or anginal pain. The symptoms are absent at rest. Home oxygen therapy is unnecessary unless there are other comorbid conditions. Diuretics mobilize edematous fluid, act on the kidneys to promote excretion of sodium and water, and reduce preload and pulmonary venous pressure. Dietary restriction of sodium aids in reducing edema. ACE inhibitors block the conversion of angiotensin I to the vasoconstrictor angiotensin II, prevent the degradation of bradykinin and other vasodilatory prostaglandins, and increase plasma renin levels and reduce aldosterone levels. The net result is systemic vasodilation, reduced systemic vascular resistance, and improved cardiac output. ➧ Test-taking Tip:The key phrase "needs additional teaching" indicates that this is a false-response item. Select the option that is incorrect for treating class II heart failure.

51. A nurse teaches a 55-year-old strict vegetarian that, to decrease the risk of developing megaloblastic anemia, the client should: 1. undergo a Schilling test. 2. increase intake of foods high in iron. 3. supplement the diet with vitamin B12. 4. have a monthly hemoglobin level drawn.

ANSWER: 3 Megaloblastic anemia is caused by deficiency of vitamin B12 or folic acid. A vegetarian can prevent a deficiency with oral vitamin supplements or fortified soy milk. The U.S. Department of Agriculture's Dietary Guidelines for Americans 2005 also recommend that people over age 50, whether or not they are vegetarian, consume vitamin B12 in its crystalline form (i.e., fortified foods or supplements). The Schilling test is used to diagnose vitamin B12 deficiency. Consuming foods high in iron will prevent iron-deficiency anemia. Monthly laboratory work is unnecessary and costly. ➧ Test-taking Tip: Focus on the issue, decrease the risk, and use the process of elimination. Eliminate options 1 and 4 because these do not decrease risk. Differentiate between iron deficiency and megaloblastic anemia to select the correct answer, option 3.

A nurse teaches a 55-year-old strict vegetarian that, to decrease the risk of developing megaloblastic anemia, the client should: 1. undergo a Schilling test. 2. increase intake of foods high in iron. 3. supplement the diet with vitamin B12. 4. have a monthly hemoglobin level drawn.

ANSWER: 3 Megaloblastic anemia is caused by deficiency of vitamin B12 or folic acid. A vegetarian can prevent a deficiency with oral vitamin supplements or fortified soy milk. The U.S. Department of Agriculture's Dietary Guidelines for Americans 2005 also recommend that people over age 50, whether or not they are vegetarian, consume vitamin B12 in its crystalline form (i.e., fortified foods or supplements). The Schilling test is used to diagnose vitamin B12 deficiency. Consuming foods high in iron will prevent iron-deficiency anemia. Monthly laboratory work is unnecessary and costly. ➧ Test-taking Tip: Focus on the issue, decrease the risk, and use the process of elimination. Eliminate options 1 and 4 because these do not decrease risk. Differentiate between iron deficiency and megaloblastic anemia to select the correct answer, option 3.

76. A nurse is caring for a client admitted with fluid volume overload. The client is receiving diuretic therapy with a loop diuretic. The potassium levels are illustrated in the chart below. On which day should the nurse expect an order for potassium replacement? Day 1 Day 2 Day 3 Day 4 5.6 mEq/L 4.4 mEq/L 3.5mEq/L 3.1 mEq/L 1. Day 1 2. Day 2 3. Day 3 4. Day 4

ANSWER: 3 Normal potassium level is 3.5 to 5.0 mEq/L. On day 3, the client is on the low end of normal. Because the client's serum potassium level is decreasing and the client is taking a diuretic, supplementation is needed to prevent a reduction of serum potassium levels below normal. The value on day 1 is high and would not require replacement. The value on day 2 is in the midrange of normal. The value on day 4 is low and would require replacement if replacement were not started on day 3.

A nurse is caring for a client admitted with fluid volume overload. The client is receiving diuretic therapy with a loop diuretic. The potassium levels are illustrated in the chart below. On which day should the nurse expect an order for potassium replacement? Day 1 Day 2 Day 3 Day 4 5.6 mEq/L 4.4 mEq/L 3.5 mEq/L 3.1 mEq/L 1. Day 1 2. Day 2 3. Day 3 4. Day 4

ANSWER: 3 Normal potassium level is 3.5 to 5.0 mEq/L. On day 3, the client is on the low end of normal. Because the client's serum potassium level is decreasing and the client is taking a diuretic, supplementation is needed to prevent a reduction of serum potassium levels below normal. The value on day 1 is high and would not require replacement. The value on day 2 is in the midrange of normal. The value on day 4 is low and would require replacement if replacement were not started on day 3. ➧ Test-taking Tip: Recall that a loop diuretic will decrease serum potassium levels. Consider this when selecting an option.

client with a diagnosis of Guillain-Barré syndrome is scheduled to receive plasmapheresis treatments. A nurse explains to the client's spouse that the purpose of plasmapheresis is to: 1. remove excess fluid from the bloodstream. 2. restore protein levels in the blood. 3. remove circulating antibodies from the bloodstream. 4. infuse lipoproteins to restore the myelin sheath.

ANSWER: 3 Plasmapheresis is a procedure in which the circulating antibodies are removed from the blood. During the procedure, blood is removed from the client, the plasma is separated, and blood cells without the plasma are returned to the client during the procedure. The purpose does not involve removing excess fluid, restoring protein, or infusing lipoproteins. ➧ Test-taking Tip: Use the process of elimination. Eliminate option 4 because plasma is associated with the blood. Apheresis means separation, thus eliminate option 2. Of the remaining two options, think about the etiology of Guillain- Barré syndrome. Eliminate option 1 because research suggests that Guillain-Barré syndrome is a cell-mediated immunological reaction.

87. A physician orders 10 mEq of potassium chloride (KCL) intravenously (IV) for an adult client whose serum potassium level prior to surgery is 3.3 mEq/L. Which action should be the nurse's first priority? 1. Administering the medication as soon as possible through IV bolus 2.Waiting to start the medication until the client is asleep in surgery 3. Diluting the medication in 100 mL of saline and administer over an hour 4. Applying warm towels to the client's arm to prevent vein irritation during administration

ANSWER: 3 Potassium chloride must be diluted prior to administration as a preventative measure since concentrated doses or rapid infusion can cause hyperkalemia, cardiac arrhythmias, or arrest. Administration of concentrated potassium chloride can cause cardiac arrhythmias or arrest. A low serum potassium level increases the client's risk of dysrhythmias during anesthesia administration. The client's serum potassium level should be within the normal range prior to surgery. Hot packing may provide comfort since the medication is irritating to the veins, but this is not most important.

A registered nurse (RN) is informed by a nursing assistant (NA) that a client, hospitalized last evening with chest pain, plans to leave right now because the pain is gone and "nobody has done anything anyway." Which is the nurse's best action? 1. Thank the NA for the information and then call the client's doctor regarding the situation 2. Tell the NA that the client has the right to leave and send the NA to help the client pack 3. Talk with the client to discuss the client's concerns and explain the plan of care 4. Tell the NA to inform the client that it is unsafe to leave and that the RN will review the test results with the client shortly

ANSWER: 3 Seeing the client provides an opportunity for further assessment and client teaching. The nurse's responsibility is to inform clients of the status of their care. Unjustifiable detention is false imprisonment. The client has a right to leave. Sending the NA to assist the client to pack or to speak to the client is inappropriate delegation of the nurse's responsibilities. Telling the client it is unsafe to leave does not explain why the client should remain in the hospital. Calling the physician is premature. ➧ Test-taking Tip: Focus on the option that uses therapeutic communication techniques and correct decision making regarding the RN's responsibility.

A nurse admits a client to a telemetry unit and obtains the following electrocardiogram (ECG) strip of the client's heart rhythm. What should be the nurse's interpretation of this rhythm strip? 1. Atrial flutter 2. Atrial fibrillation 3. Sinus bradycardia 4. Sinus rhythm with premature atrial contractions (PACs)

ANSWER: 3 Sinus bradycardia is a regular rhythm with a ventricular rate less than 60 beats per minute (bpm), and one discernable P wave prior to each QRS. Atrial flutter is either a regular or an irregular rhythm with multiple discernable P waves prior to each QRS complex and no measurable PR interval. Atrial fibrillation is an irregular rhythm with multiple nondiscernable, fibrillatory P waves prior to each QRS and no measurable PR interval. Sinus rhythm with PACs is an irregular rhythm with a ventricular rate between 60 and 100 bpm, one discernable P wave prior to each QRS, and a PR interval between 0.12 and 0.20 seconds, with the presence of premature atrial beats that occur early in the cardiac cycle. The PACs also have one discernable P wave prior to each QRS and a PR interval between 0.08 and 0.20 seconds. ➧ Test-taking Tip: Use the steps in interpreting an ECG rhythm to select the correct option. Note that the rhythm is regular, so eliminate option 4, which is an irregular rhythm. Recall that atrial fibrillation and atrial flutter do not have a measurable PR interval, so eliminate options 1 and 2.

A hospital nurse is teaching coworkers how to prevent varicose veins. Which recommendation by the nurse is most accurate? 1. Wear low-heeled comfortable shoes 2. Move your legs back and forth often 3.Wear support hose or thromboembolic deterrent stockings (TEDS). 4.Wear clean, white cotton socks with tennis shoes

ANSWER: 3 Support or compression hose help decrease edema and increase circulation back to the heart, thereby preventing varicose veins. Low-heeled shoes and clean white cotton socks with tennis shoes will decrease foot discomfort but will not prevent varicose veins. Leg movement helps prevent deep vein thrombosis, but not varicose veins.

A hospital nurse is teaching coworkers how to prevent varicose veins. Which recommendation by the nurse is most accurate? 1. Wear low-heeled comfortable shoes 2. Move your legs back and forth often 3.Wear support hose or thromboembolic deterrent stockings (TEDS). 4.Wear clean, white cotton socks with tennis shoes

ANSWER: 3 Support or compression hose help decrease edema and increase circulation back to the heart, thereby preventing varicose veins. Low-heeled shoes and clean white cotton socks with tennis shoes will decrease foot discomfort but will not prevent varicose veins. Leg movement helps prevent deep vein thrombosis, but not varicose veins. ➧ Test-taking Tip: Use the process of elimination, ruling out options 1 and 4 because they address the feet. Eliminate option 2 after noting that back-and-forth movement of the legs is not the same as dorsiflexion and plantar flexion and will not promote return of blood to the heart.

57. The family of a client who is scheduled for emergency surgery following an accident asks if they can donate blood for the client. The client's blood type is B negative. A nurse informs the family that packed red blood cells (PRBCs) could likely be used from family members whose blood type is: 1. type A positive. 2. type B positive. 3. type B negative. 4. type O positive. 5. type O negative. 6. type AB positive.

ANSWER: 3 The ABO system identifies the type of antigen present on the person's erythrocyte membrane, A, B, both A and B, or neither A or B (type O). The person with B negative blood type has B antigen on the erythrocyte and does not have an Rh (or D) antigen on the cell (whereas Rh positive does have a D antigen on the cell). The person can only receive PRBC of the same blood type (option 3) to prevent a significant PRBC reaction. Formerly in emergency situations, it was thought that O blood type was safe to administer when the client's blood type was not known. The American Red Cross now advises against this practice and recommends only administering blood of the same blood type.

73. A nurse is caring for a client with heart failure who has been placed on a 2,000-mL fluid restriction. The nurse is responsible for establishing a plan for how that restriction should be distributed over a 24-hour period. Which plan, developed by the nurse, is best? Shift/Time 7-3 3-11 11-7 1. 1,000 mL 1,000 mL 0 mL 2. 900 mL 900 mL 200 mL 3. 1,000 mL 700 mL 300 mL 4. 700 mL 700 mL 600 mL

ANSWER: 3 The general rule is to provide half of the total restriction during the day and the other half between evening and nights, with most fluids offered in the evening. Option 1 is incorrect because fluids should be available during the night. Options 2 and 4 provide for a large amount of fluid intake just before bedtime; this should be avoided because it disrupts sleep.

nurse is caring for a client with heart failure who has been placed on a 2,000-mL fluid restriction. The nurse is responsible for establishing a plan for how that restriction should be distributed over a 24-hour period. Which plan, developed by the nurse, is best? Shift/Time 7-3 3-11 11-7 1. 1,000 mL 1,000 mL 0 mL 2. 900 mL 900 mL 200 mL 3. 1,000 mL 700 mL 300 mL 4. 700 mL 700 mL 600 mL

ANSWER: 3 The general rule is to provide half of the total restriction during the day and the other half between evening and nights, with most fluids offered in the evening. Option 1 is incorrect because fluids should be available during the night. Options 2 and 4 provide for a large amount of fluid intake just before bedtime; this should be avoided because it disrupts sleep. ➧ Test-taking Tip: Think about your own fluid intake and when you likely consume the most amount of fluid. Recall that there are usually two meals provided during a 7 a.m.-to 3-p.m. period that would increase fluid intake during this time period.

67. A nurse is interpreting an ECG rhythm strip for a 2-year-old child with heart failure secondary to a congenital heart defect. In analyzing the rhythm, the nurse notes the measurements of PR interval is 0.26 seconds, the QRS is 0.08 seconds, and the QT is 0.28. The ventricular rate is 126 bpm. A nurse interprets the rhythm as: 1. sinus bradycardia. 2. sinus rhythm with a bundle branch block. 3. sinus rhythm with a first-degree AV block. 4. sinus tachycardia with a first-degree AV block.

ANSWER: 3 The normal heart rate for a 2-year-old is 80 to 130 bpm. A normal PR interval measures 0.12 to 0.20 seconds. The QRS is normal (0.6 to 0.10 seconds), and the QT is rate dependent. If the rate is fast, the QT will be shorter. It is within the normal range for the ventricular rate. The ventricular rate in sinus bradycardia for a 2-year-old should be less than 80. In a bundle branch block, the QRS interval should be greater than or equal to 0.12 seconds. In sinus tachycardia, the ventricular rate should be greater than 130 bpm for a 2-year-old. ➧ Test-taking Tip: In order to answer this question,

A nurse is interpreting an ECG rhythm strip for a 2-year-old child with heart failure secondary to a congenital heart defect. In analyzing the rhythm, the nurse notes the measurements of PR interval is 0.26 seconds, the QRS is 0.08 seconds, and the QT is 0.28. The ventricular rate is 126 bpm. A nurse interprets the rhythm as: 1. sinus bradycardia. 2. sinus rhythm with a bundle branch block. 3. sinus rhythm with a first-degree AV block. 4. sinus tachycardia with a first-degree AV block.

ANSWER: 3 The normal heart rate for a 2-year-old is 80 to 130 bpm. A normal PR interval measures 0.12 to 0.20 seconds. The QRS is normal (0.6 to 0.10 seconds), and the QT is rate dependent. If the rate is fast, the QT will be shorter. It is within the normal range for the ventricular rate. The ventricular rate in sinus bradycardia for a 2-year-old should be less than 80. In a bundle branch block, the QRS interval should be greater than or equal to 0.12 seconds. In sinus tachycardia, the ventricular rate should be greater than 130 bpm for a 2-year-old. ➧ Test-taking Tip: In order to answer this question, knowledge of the normal ECG waveforms and the measurements is necessary. If this is unknown, then begin to eliminate options. The heart rate is normal for a 2-year-old. Eliminate options 1 and 4. This increases the chance of getting a right answer to 50%.

aPTT (sec) <50 50 - <60 60 - <86 86 - <96 96 - 120 >120 IV Bolus (units) 5,000 0 0 0 0 0 Hold (minutes) 0 0 0 0 30 60 IV Rate (mL/hr) 31 28 25 23 21 17 Repeat aPTT/PT 6 hr 6 hr Next AM Next AM 6 hr 6 hR 1. Start the heparin at 25 mL/hr because that is the initiation rate on the protocol. 2. Bolus with 5,000 units and begin the drip at 31 mL/hr because that is the first line of the protocol. 3. Contact the surgeon to determine the rate at which the heparin drip should be started because the order is not clear. 4. Start the heparin at 28 mL/hr per protocol and obtain an activated partial thromboplastin time (aPTT) in 6 hours to reestablish the baseline.

ANSWER: 3 The order is not clear as to what rate to start the heparin. Use critical- thinking skills to evaluate the orders. Determine which physician is managing the heparin protocol. Continue to ask questions until the orders are completely clarified. If it is change of shift and the orders have not been clarified, make sure the handoff communication includes the need to clarify orders before administration of heparin. Never guess or assume what the physician meant to write. Nurses can expect clearly written orders. ➧Test-taking Tip: Carefully read the directions from the physicians. Note there is no clear direction. #27

A client admitted with a diagnosis of acute coronary syndrome calls for a nurse after experiencing sharp chest pains that radiate to the left shoulder. The nurse notes, prior to entering the client's room, that the client's rhythm is sinus tachycardia with a 10-beat run of premature ventricular contractions (PVCs). Admitting orders included all of the following interventions for treating chest pain. Which should the nurse implement first? 1. Obtain a stat 12-lead electrocardiogram (ECG). 2. Administer IV nitroglycerin. 3. Administer sublingual nitroglycerin. 4. Administer morphine sulfate intravenously.

ANSWER: 3 Though a stat 12-lead ECG is needed to identify ischemia or infarct location, the first action is to treat the client. Sublingual nitroglycerin dilates coronary arteries and will enhance blood flow to the myocardium. Once oxygen is in place and the vital signs known, nitroglycerin should be administered. Morphine sulfate is a narcotic analgesic used for pain control and anxiety reduction. Because it is a controlled substance, extra steps are needed to retrieve the medication from a secure source, so this is not the first action.

A nurse who is beginning a shift on a cardiac stepdown unit receives shift report for four clients. In which order should the nurse assess the clients? Prioritize the nurse's actions by placing each client in order from most urgent (1) to least urgent (4). ______ A 56-year-old client who was admitted 1 day ago with chest pain receiving intravenous (IV) heparin and has a partial thromboplastin time (PTT) due back in 30 minutes ______ A 62-year-old client with end-stage cardiomyopathy, blood pressure (BP) of 78/50 mm Hg, 20 mL/hr urine output, and a "Do Not Resuscitate" order and whose family has just arrived ______ A 72-year-old client who was transferred 2 hours ago from the intensive care unit (ICU) following a coronary artery bypass graft and has new onset atrial fibrillation with rapid ventricular response ______ A 38-year-old postoperative client who had an aortic valve replacement 2 days ago, BP 114/72 mm Hg, heart rate (HR) 100 beats/min, respiratory rate (RR) 28 breaths/min, and temperature 101.2°F (38.4°C)

ANSWER: 3, 4, 1, 2 The client with new onset atrial fibrillation should be assessed first because it is the most life threatening. The postoperative client with the elevated temperature should be assessed next because the elevated temperature, RR, and HR increase the demands on the heart and could be a sign of pulmonary complications. Third, the nurse should assess the client with the heparin infusion. PTT results should be back, and the dose may require adjustment. Last, the client with end-stage cardiomyopathy should be assessed. The family will have had time alone with the client, and the client and family may need emotional support. ➧ Test-taking Tip: When establishing priorities, first determine life-threatening situations and then prioritize remaining clients by using the ABCs (airway, breathing, and circulation). Recall from Maslow's Hierarchy of Needs that physiological problems are priority over psychosocial issues, thus the client with endstage cardiomyopathy should be assessed last.

Blood for cardiac enzymes and serum laboratory tests are drawn on a diabetic client admitted to an emergency department (ED) 5 hours after beginning to experience chest pressure. A nurse reviews the following laboratory results. Which serum laboratory findings should the nurse report to a primary healthcare provider (HCP) immediately due to the possibility that the client may be experiencing a myocardial infarction (MI)? SELECT ALL THAT APPLY. BUN (10-20 mg/dL) 30 SCr (0.4-1.4 mg/dL) 1.8 Ca (8.5-10.5 mg/dL) 9.0 Cl (100-108 mEq/L) 105 CO2 (25-29 mEq/L) 24 Glucose (70-110 mg/dL) 160 Na (136-146 mEq/L) 135 K (3.8-5.3 mEq/L) 5.8 Mag (1.7-2.2 mg/dL) 1.6 CK (0-160 u/L) 320 CK-MB (0-16 u/L) 32 Troponin T (cTnT) 34 (0.0-0.4 NG/mL) WBC (3.9-11.9 K/μL) 14 RBC (4.08-5.79 m/μL) 5.0 Hgb (13.1-17.1%) 15 Hct (38.7-51 g/dL) 48 Platelets (PLT) (179-450 K/μL) 175 PT (9.2-11.9 sec)/INR (0.9-1.1 sec) 22/2.2 APTT (24-33 sec) 66 1. SCr 2. PT/INR 3. CK 4. CK-MB 5. Platelets 6. Troponin T

ANSWER: 3, 4, 6 A diabetic client may not experience the typical chest pain of a MI. Three to 12 hours after an MI the CK levels begin to rise, then peak in 24 hours, and return to normal within 2 to 3 days. The CK-MB band is specific to myocardial cells. Cardio-specific troponins (troponin T, cTnT, and troponin I, cTnI) are released into circulation after myocardial injury, are highly specific indicators of MI, and have greater sensitivity and specificity than CK-MB. SCr means serum creatinine, which is used to determine renal function and the kidneys' abilities to excrete creatinine. PT/INR (prothrombin time/international normalized ratio) is used to measure blood coagulation, especially with the use of warfarin (Coumadin®). Platelets are necessary for blood coagulation. ➧ Test-taking Tip:Though all abnormal values should be reported to the HCP, the issue of the question is laboratory values specific for MI. Only select the options that pertain to MI.

Blood for cardiac enzymes and serum laboratory tests are drawn on a diabetic client admitted to an emergency department (ED) 5 hours after beginning to experience chest pressure. A nurse reviews the following laboratory results. Which serum laboratory findings should the nurse report to a primary healthcare provider (HCP) immediately due to the possibility that the client may be experiencing a myocardial infarction (MI)? SELECT ALL THAT APPLY. 1. SCr 2. PT/INR 3. CK 4. CK-MB 5. Platelets 6. Troponin T

ANSWER: 3, 4, 6 A diabetic client may not experience the typical chest pain of a MI. Three to 12 hours after an MI the CK levels begin to rise, then peak in 24 hours, and return to normal within 2 to 3 days. The CK-MB band is specific to myocardial cells. Cardio-specific troponins (troponin T, cTnT, and troponin I, cTnI) are released into circulation after myocardial injury, are highly specific indicators of MI, and have greater sensitivity and specificity than CK-MB. SCr means serum creatinine, which is used to determine renal function and the kidneys' abilities to excrete creatinine. PT/INR (prothrombin time/international normalized ratio) is used to measure blood coagulation, especially with the use of warfarin (Coumadin®). Platelets are necessary for blood coagulation. ➧ Test-taking Tip:Though all abnormal values should be reported to the HCP, the issue of the question is laboratory values specific for MI. Only select the options that pertain to MI.

A nurse is instructing a client diagnosed with coronary artery disease about care at home. The nurse determines that teaching is effective when the client states: SELECT ALL THAT APPLY. 1. "If I have chest pain, I should contact my physician immediately." 2. "I should carry my nitroglycerin in my front pants pocket so it is handy." 3. "If I have chest pain, I stop activity and place one nitroglycerin tablet under my tongue." 4. "I should always take three nitroglycerin tablets, 5 minutes apart." 5. "I plan to avoid being around people when they are smoking." 6. "I plan on walking on most days of the week for at least 30 minutes."

ANSWER: 3, 5, 6

nurse is instructing a client diagnosed with coronary artery disease about care at home. The nurse determines that teaching is effective when the client states: SELECT ALL THAT APPLY. 1. "If I have chest pain, I should contact my physician immediately." 2. "I should carry my nitroglycerin in my front pants pocket so it is handy." 3. "If I have chest pain, I stop activity and place one nitroglycerin tablet under my tongue." 4. "I should always take three nitroglycerin tablets, 5 minutes apart." 5. "I plan to avoid being around people when they are smoking." 6. "I plan on walking on most days of the week for at least 30 minutes."

ANSWER: 3, 5, 6 Stopping activity decreases the body's demand for oxygen. One nitroglycerin tablet, taken sublingually, dilates the coronary arteries and increases oxygen to the myocardium. If pain is unrelieved, a second tablet should be taken 5 minutes later. Passive smoke can cause vasoconstriction and decrease blood flow velocity even in healthy young adults. The American Heart Association recommends exercising for 30 minutes on most days of the week. Medical attention is required only if pain persists and then the client should call 911 rather than the physician because emergency treatment may be necessary. Nitroglycerin loses its potency if stored in warm, moist areas, making the client's pants pocket an undesirable location for storage. If pain is relieved after one tablet, another tablet is not required. The standard dose for nitroglycerin is one tablet or spray 5 minutes apart until pain is relieved, to a maximum of three tablets. ➧ Test-taking Tip:The key words are "teaching is effective." Select the client statements that are correct.

A nurse is instructing a client diagnosed with coronary artery disease about care at home. The nurse determines that teaching is effective when the client states: SELECT ALL THAT APPLY. 1. "If I have chest pain, I should contact my physician immediately." 2. "I should carry my nitroglycerin in my front pants pocket so it is handy." 3. "If I have chest pain, I stop activity and place one nitroglycerin tablet under my tongue." 4. "I should always take three nitroglycerin tablets, 5 minutes apart." 5. "I plan to avoid being around people when they are smoking." 6. "I plan on walking on most days of the week for at least 30 minutes."

ANSWER: 3, 5, 6 Stopping activity decreases the body's demand for oxygen. One nitroglycerin tablet, taken sublingually, dilates the coronary arteries and increases oxygen to the myocardium. If pain is unrelieved, a second tablet should be taken 5 minutes later. Passive smoke can cause vasoconstriction and decrease blood flow velocity even in healthy young adults. The American Heart Association recommends exercising for 30 minutes on most days of the week. Medical attention is required only if pain persists and then the client should call 911 rather than the physician because emergency treatment may be necessary. Nitroglycerin loses its potency if stored in warm, moist areas, making the client's pants pocket an undesirable location for storage. If pain is relieved after one tablet, another tablet is not required. The standard dose for nitroglycerin is one tablet or spray 5 minutes apart until pain is relieved, to a maximum of three tablets. ➧ Test-taking Tip:The key words are "teaching is effective." Select the client statements that are correct.

A nurse notes that a client, who experienced a myocardial infarction (MI) 3 days ago, seems unusually fatigued. Upon assessment, the nurse finds that the client is dyspneic with activity, has a heart rate (HR) of 110 beats per minute (bpm), and has generalized edema. Which action by the nurse is most appropriate? 1. Administer high-flow oxygen 2. Encourage the client to rest more 3. Continue to monitor the client's heart rhythm 4. Compare the client's admission weight with the client's current weight .

ANSWER: 4 A complication of MI is heart failure. Signs of heart failure include fatigue, dyspnea, tachycardia, edema, and weight gain. Other signs include nocturia, skin changes, behavioral changes, and chest pain. There is no indication that the client is hypoxic and in need of high-flow oxygen. To treat the dyspnea, oxygen by nasal cannula would be appropriate. The fatigue is caused by decreased cardiac output, impaired perfusion to vital organs, decreased tissue oxygenation, and anemia. Rest alone will not relieve the fatigue. Interventions are needed to improve cardiac output and tissue oxygenation. A heart rate of 110 bpm suggests tachycardia; the symptoms together imply heart failure. Further data collection is needed to confirm the findings. Continuing to monitor the client's heart rhythm, without further assessment, will delay an appropriate intervention. ➧ Test-taking Tip: Use the nursing process to determine the next action. Before a conclusion can be reached, additional data are needed. The nurse should complete the assessment process. Eliminate options 1 and 2 because these are interventions. Of options 3 and 4 determine which option would provide the most immediate information to make a conclusion about the data

A nurse notes that a client, who experienced a myocardial infarction (MI) 3 days ago, seems unusually fatigued. Upon assessment, the nurse finds that the client is dyspneic with activity, has a heart rate (HR) of 110 beats per minute (bpm), and has generalized edema. Which action by the nurse is most appropriate? 1. Administer high-flow oxygen 2. Encourage the client to rest more 3. Continue to monitor the client's heart rhythm 4. Compare the client's admission weight with the client's current weight

ANSWER: 4 A complication of MI is heart failure. Signs of heart failure include fatigue, dyspnea, tachycardia, edema, and weight gain. Other signs include nocturia, skin changes, behavioral changes, and chest pain. There is no indication that the client is hypoxic and in need of high-flow oxygen. To treat the dyspnea, oxygen by nasal cannula would be appropriate. The fatigue is caused by decreased cardiac output, impaired perfusion to vital organs, decreased tissue oxygenation, and anemia. Rest alone will not relieve the fatigue. Interventions are needed to improve cardiac output and tissue oxygenation. A heart rate of 110 bpm suggests tachycardia; the symptoms together imply heart failure. Further data collection is needed to confirm the findings. Continuing to monitor the client's heart rhythm, without further assessment, will delay an appropriate intervention. ➧ Test-taking Tip: Use the nursing process to determine the next action. Before a conclusion can be reached, additional data are needed. The nurse should complete the assessment process. Eliminate options 1 and 2 because these are interventions. Of options 3 and 4 determine which option would provide the most immediate information to make a conclusion about the data.

When assessing the cardiovascular system of a 75-year-old male, a nurse auscultates a systolic heart murmur. This is the only abnormality noted. Which analysis by the nurse is correct? 1. Usually representative of some kind of underlying heart disease 2. Indication for valve replacement 3. Indication that the client has congestive heart failure (CHF) 4. Common due to age-related calcification and stiffening of the heart valves

ANSWER: 4 Age-related cardiovascular changes include calcification and stiffening of the heart valves (aortic and mitral), causing incomplete closure (systolic murmur). Approximately 60% of older adults have murmurs. Murmurs are abnormal, vibratory heart sounds. Other causes, including rheumatic fever and congenital defects of the valves, should not be the first consideration. More cardiac symptoms would need to be present to consider valve replacement. Following valve replacement, a murmur usually is auscultated.

A nurse assesses a client who has just returned to a telemetry unit after having a coronary angiogram using the left femoral artery approach. The client's baseline blood pressure (BP) during the procedure was 130/72 mm Hg and the cardiac rhythm was a normal sinus throughout. Which assessment finding should indicate to the nurse that the client may be experiencing a complication? 1. BP 144/78 mm Hg 2. Pedal pulses palpable at +1 3. Left groin soft with 1 cm ecchymotic area 4. Apical pulse 132 beats per minute (bpm) with an irregular-irregular rhythm ng.

ANSWER: 4 An apical pulse of 132 (bpm) with an irregular-irregular rhythm could indicate atrial fibrillation or a rhythm with premature beats. Dysrhythmias are a complication that can occur following coronary angiogram. The client should be placed on a cardiac monitor to determine the rhythm. A slight elevation of blood pressure could be related to pain at the incision site. It is not indicative of a complication without additional data. Usually pulses are palpable at +2, but without additional baseline data on the clients' pulses, this warrants monitoring but is not indicative in itself of a complication. A soft groin area where the puncture site is located is a normal finding. Ecchymosis (bruising) does not indicate a complication. ➧ Test-taking Tip: Think about the potential complications that can occur after a coronary angiogram. Review each option to determine if the findings suggest a complication. Select the option that would be the most abnormal findiNG

A nurse assesses a client who has just returned to a telemetry unit after having a coronary angiogram using the left femoral artery approach. The client's baseline blood pressure (BP) during the procedure was 130/72 mm Hg and the cardiac rhythm was a normal sinus throughout. Which assessment finding should indicate to the nurse that the client may be experiencing a complication? 1. BP 144/78 mm Hg 2. Pedal pulses palpable at 1 3. Left groin soft with 1 cm ecchymotic area 4. Apical pulse 132 beats per minute (bpm) with an irregular-irregular rhythm

ANSWER: 4 An apical pulse of 132 (bpm) with an irregular-irregular rhythm could indicate atrial fibrillation or a rhythm with premature beats. Dysrhythmias are a complication that can occur following coronary angiogram. The client should be placed on a cardiac monitor to determine the rhythm. A slight elevation of blood pressure could be related to pain at the incision site. It is not indicative of a complication without additional data. Usually pulses are palpable at 2, but without additional baseline data on the clients' pulses, this warrants monitoring but is not indicative in itself of a complication. A soft groin area where the puncture site is located is a normal finding. Ecchymosis (bruising) does not indicate a complication.

A nurse is preparing to perform an electrocardiogram (ECG) on several pediatric clients. Which client would not benefit from an ECG? 1. A 4-year-old with tachycardia 2. A 3-year-old with bradycardia 3. A 10-year-old with an irregular pulse 4. An infant with a splitting of the S2 heart sound only when the infant takes a deep breath

ANSWER: 4 An infant with a splitting of the S2 heart sound when the child takes a deep breath would not benefit from an ECG because it is a normal finding in infants and young children. ECG is useful in diagnosing tachycardia, bradycardia, and irregular pulses. These rhythms can be recorded through an ECG. ➧ Test-taking Tip: Note that three options are similar and one is different. Often the option that is different is the answer.

A client admitted to a telemetry unit with a diagnosis of Prinzmetal's angina, has the following medications ordered. Upon interpretation of the client's electrocardiogram (ECG) rhythm, the nurse notes a prolonged PR interval of 0.32 second. Based on this information, which medication order should the nurse question administering to the client? 1. Isosorbide mononitrate (Imdur®) 20 mg oral daily upon awakening 2. Amlodipine (Norvasc®) 10 mg oral daily 3. Nitroglycerin (Nitrostat®) 0.4 mg sublingual prn for chest pain 4. Atenolol (Tenormin®) 50 mg oral daily.

ANSWER: 4 Atenolol, a beta-blocker, blocks stimulation of beta1 (myocardial)- adrenergic receptors, causing a reduction in blood pressure and heart rate. A side effect of the medication is a prolongation of the PR interval (normal PR interval is 0.12 to 0.20 second). Continued use of the drug can result in heart block. Nitrates and calcium channel blockers (CCBs) are the mainstays of medical therapy for variant angina rather than beta blockers. Isosorbide mononitrate, a nitrate, causes vasodilatation of the large coronary arteries. Nitrates act as an exogenous source of nitric oxide, which causes vascular smooth muscle relaxation resulting in a decrease in myocardial oxygen consumption and may have a modest effect on platelet aggregation and thrombosis. Amlodipine, a CCB, relaxes coronary smooth muscle and produces coronary vasodilation, which in turn improves myocardial oxygen delivery. Nitroglycerin sublingual effectively treats episodes of angina and myocardial ischemia within minutes of administration, and the long-acting nitrate preparation (Imdur®) reduces the frequency of recurrent events. ➧ Test-taking Tip: The focus of the question is a medication contraindicated with a prolonged PR interval. Think about the action of the medications prescribed. Recall that Prinzmetal's angina is caused from coronary artery vasospasm and that nitrates and CCBs are the medications of choice for treatment. Both the prolonged PR interval and the diagnosis of Prinzmetal's angina are clues to selecting the correct option.

A nurse collects the following assessment data on a client who has no known health problems: blood pressure (BP) 135/89 mm Hg; body mass index (BMI) 23; waist circumference 34 inches; serum creatinine 0.9 mg/dL; serum K 4.0 mEq/L; low-density lipoprotein (LDL) cholesterol 200 mg/dL; high-density lipoprotein (HDL) cholesterol 25 mg/dL; and triglycerides 180 mg/dL. Which order from the client's health-care provider should the nurse anticipate? 1. 1,500-calorie regular diet. 2. No added salt, low saturated fat, low-potassium diet. 3. Hydrochlorothiazide (HydroDIURIL®) 25 mg twice daily. 4. Atorvastatin (Lipitor®) 20 mg daily.

ANSWER: 4 Atorvastatin is used to manage hypercholesterolemia. It lowers the total serum LDL cholesterol and triglycerides and slightly increases HDL cholesterol by inhibiting 3-hydroxy-3-methylglutaryl-coenzyme A (HMG-CoA) reductase, an enzyme that is responsible for catalyzing an early step in the synthesis of cholesterol. For persons with 0-1 risk factors, the goal for LDL is less than 160 mg/dL (4.14 mmol/L), and drug therapy is considered when LDL is greater than or equal to 190 mg/dL (4.91 mmol/L). Normal triglycerides are 40 to 150 mg/dL (0.45-1.69 mmol/L). A low-calorie diet is not indicated. The normal BMI is 18.5 to 24.9. While a low-saturated-fat diet in option 2 is indicated, a low-potassium diet is not because the serum K of 4.0 mEq/L is normal. The client's BP is slightly elevated but would be initially treated with lifestyle changes, not a diuretic. ➧ Test-taking Tip: Focus on the data provided in the situation and identify the abnormal findings. If unable to identify the abnormal data because of lack of knowledge of normal lab values, note that the client's serum cholesterol level analysis includes more data than other problems. Conclude that these are abnormal data and then use "lipids" as a key to identifying the correct option. Review laboratory values and cardiac medications if you had difficulty with this question.

A nurse collects the following assessment data on a client who has no known health problems: blood pressure (BP) 135/89 mm Hg; body mass index (BMI) 23; waist circumference 34 inches; serum creatinine 0.9 mg/dL; serum K 4.0 mEq/L; low-density lipoprotein (LDL) cholesterol 200 mg/dL; high-density lipoprotein (HDL) cholesterol 25 mg/dL; and triglycerides 180 mg/dL. Which order from the client's health-care provider should the nurse anticipate? 1. 1,500-calorie regular diet. 2. No added salt, low saturated fat, low-potassium diet. 3. Hydrochlorothiazide (HydroDIURIL®) 25 mg twice daily. 4. Atorvastatin (Lipitor®) 20 mg daily.

ANSWER: 4 Atorvastatin is used to manage hypercholesterolemia. It lowers the total serum LDL cholesterol and triglycerides and slightly increases HDL cholesterol by inhibiting 3-hydroxy-3-methylglutaryl-coenzyme A (HMG-CoA) reductase, an enzyme that is responsible for catalyzing an early step in the synthesis of cholesterol. For persons with 0-1 risk factors, the goal for LDL is less than 160 mg/dL (4.14 mmol/L), and drug therapy is considered when LDL is greater than or equal to 190 mg/dL (4.91 mmol/L). Normal triglycerides are 40 to 150 mg/dL (0.45-1.69 mmol/L). A low-calorie diet is not indicated. The normal BMI is 18.5 to 24.9. While a low-saturated-fat diet in option 2 is indicated, a low-potassium diet is not because the serum K of 4.0 mEq/L is normal. The client's BP is slightly elevated but would be initially treated with lifestyle changes, not a diuretic. ➧ Test-taking Tip: Focus on the data provided in the situation and identify the abnormal findings. If unable to identify the abnormal data because of lack of knowledge of normal lab values, note that the client's serum cholesterol level analysis includes more data than other problems. Conclude that these are abnormal data and then use "lipids" as a key to identifying the correct option. Review laboratory values and cardiac medications if you had difficulty with this question.

93. Which action should a nurse take when caring for a client with thrombocytopenia? 1. Teach the client to use dental floss after teeth brushing to prevent dental caries 2. Treat an elevated temperature of 102°F (39°C) or greater with two 325-mg aspirin tablets 3. Prevent injury by maintaining the client on strict bedrest 4. Assess for signs of blurred vision, headache, or mental status changes

ANSWER: 4 Bleeding can occur with low platelet levels (usually less than 50,000/mm3 or 50 K/μL). Blurred vision, headache, or mental status changes can be signs of intracranial bleeding. Because the client is prone to bleeding, dental flossing and aspirin are contraindicated. A client should be ambulated with assistance.

83. A nurse is counseling a client with cardiac disease who has limited food refrigeration capabilities and prefers using canned vegetables. Which nutrient excess should the nurse caution the client about when eating mainly canned, rather than fresh, vegetables? 1. Potassium 2. Vitamin A 3. Vitamin C 4. Sodium

ANSWER: 4 Canned vegetables, even those low in sodium, have higher sodium levels than fresh or frozen. Potassium and vitamins A and C are not a concern in the processing of canned vegetables.

A nurse is counseling a client with cardiac disease who has limited food refrigeration capabilities and prefers using canned vegetables. Which nutrient excess should the nurse caution the client about when eating mainly canned, rather than fresh, vegetables? 1. Potassium 2. Vitamin A 3. Vitamin C 4. Sodium

ANSWER: 4 Canned vegetables, even those low in sodium, have higher sodium levels than fresh or frozen. Potassium and vitamins A and C are not a concern in the processing of canned vegetables. ➧ Test-taking Tip: Key words are "excessive" and "canned." Think about how canned vegetables are preserved.

91. A nurse is caring for a client who has been transferred from the coronary care unit following a myocardial infarction (MI). The client frequently calls the nurse to check on the ECG monitor rhythm. When the cardiac rehabilitation therapist arrives to transport the client for cardiac rehabilitation exercises, the client questions if he or she is ready for activity so soon after an MI. Which behavioral response should the nurse determine that the client is exhibiting? 1. Depression 2. Denial 3. Anger 4. Dependency

ANSWER: 4 Dependency is a state of reliance on another and can be a behavioral response to acute coronary syndrome. Depression is usually manifested by symptoms of withdrawal, crying, or apathy. These behaviors are not noted in the situation. Denial is a defense mechanism that allows minimization of a threat. It may be manifested by minimizing the severity of the medical condition or ignoring activity restrictions, or avoids discussing illness or its significance. Anger is often manifested by antagonistic behaviors that may be directed at family, staff, or medical regimen.

A client tests positive for factor V Leiden (FVL). A nurse recognizes that because the genetic trait is associated with venous thromboembolism (VTE) the client is: 1. also at a greater risk for myocardial infarction. 2. more likely to be of African American heritage. 3. at risk for premature death. 4. at risk for VTE if taking estrogen as an oral contraceptive or hormone replacement.

ANSWER: 4 FVL is a genetic trait that increases risk of VTE. There is no associated risk of arterial thrombosis associated with myocardial infarction. FVL is predominantly found in Caucasians and is common in older adult clients. ➧ Test-taking Tip: Key words in the stem are "genetic trait" and "VTE." Use the process of elimination because it is well known that the risk of thromboembolism is associated with taking oral contraceptives.

An emergency department nurse is assessing a pediatric client suspected of having acute pericarditis. Which assessment finding should the nurse conclude supports the diagnosis of acute pericarditis? 1. Bilateral lower extremity pain 2. Pain on expiration 3. Pleural friction rub 4. Pericardial friction rub

ANSWER: 4 Inflammation of the pericardial sac from acute pericarditis produces a pericardial friction rub. Decreased perfusion to the extremities can cause extremity pain, but this does not occur with pericarditis. Pain on inspiration, not expiration, is present with pericarditis. The friction rub is pericardial, not pleural. ➧ Test-taking Tip: Focus on the word "pericarditis." "Peri-" is around, "cardio-" pertains to the heart, and "-itis" is inflammation. Eliminate options 1, 2, and 3 because option 4 pertains to the heart.

60. A client who has received 50 mL of a unit of whole blood complains of low back pain. In response to this client's symptom, a nurse should first: 1. reposition the client. 2. assess the pain further. 3. administer an analgesic. 4. stop the blood transfusion.

ANSWER: 4 Low back pain is a symptom of a potentially life-threatening acute hemolytic reaction. The pain is caused from agglutination of red blood cells in the kidneys and renal vasoconstriction. Hemolytic reactions occur most often within the first 50 mL of the infusion.

A client who has received 50 mL of a unit of whole blood complains of low back pain. In response to this client's symptom, a nurse should first: 1. reposition the client. 2. assess the pain further. 3. administer an analgesic. 4. stop the blood transfusion.

ANSWER: 4 Low back pain is a symptom of a potentially life-threatening acute hemolytic reaction. The pain is caused from agglutination of red blood cells in the kidneys and renal vasoconstriction. Hemolytic reactions occur most often within the first 50 mL of the infusion. ➧ Test-taking Tip: When a question asks for the first action, all other actions may also be correct, but the answer is the option that should be first. Use the nursing process. Recall that this could be a life-threatening situation, thus an action follows the initial assessment, not further assessment. Eliminate option 2. Of the remaining options, think about which action is quickest, option 4.

62. A young adult with a diagnosis of hemophilia A is receiving a monthly scheduled dose of factor VIII cryoprecipitate (Bioclate®). While a nurse is administering the Bioclate®, the client begins to cry. Which nursing response would be most appropriate? 1. "Why are you crying? You seem afraid when I am administering the Bioclate®." 2. "Is it painful when I administer the Bioclate® intravenous push? If it is, I can administer it by infusion." 3. "I know this is uncomfortable for you, but this will only take about 3 minutes to administer." 4. "If you want to talk to me about what you are feeling, I am here to listen."

ANSWER: 4 The nurse is offering self, which is a therapeutic communication technique. Option 1 challenges the client's feelings. Option 2 is seeking information, but is not most appropriate because the initial response from an adult would not be crying if it were painful. Option 3 ignores the client's feelings and presumes that the nurse knows what initiated the client's crying.

A young adult with a diagnosis of hemophilia A is receiving a monthly scheduled dose of factor VIII cryoprecipitate (Bioclate®). While a nurse is administering the Bioclate®, the client begins to cry. Which nursing response would be most appropriate? 1. "Why are you crying? You seem afraid when I am administering the Bioclate®." 2. "Is it painful when I administer the Bioclate® intravenous push? If it is, I can administer it by infusion." 3. "I know this is uncomfortable for you, but this will only take about 3 minutes to administer." 4. "If you want to talk to me about what you are feeling, I am here to listen."

ANSWER: 4 The nurse is offering self, which is a therapeutic communication technique. Option 1 challenges the client's feelings. Option 2 is seeking information, but is not most appropriate because the initial response from an adult would not be crying if it were painful. Option 3 ignores the client's feelings and presumes that the nurse knows what initiated the client's crying. ➧ Test-taking Tip: Focus on the issue, the nurse's response to the client's crying and not on the nurse's administration of the Bioclate®. The client has been receiving this monthly, so there is likely another reason for the client's crying. Eliminate options 1, 2, and 3 because they focus on the nurse's procedure.

A nurse, assessing a client hospitalized following a myocardial infarction (MI), obtains the following vital signs: blood pressure (BP) 78/38 mm Hg, heart rate (HR) 128, respiratory rate (RR) 32. For which life-threatening complication should the nurse carefully monitor the client? 1. Pulonary embolism 2. Cardiac tamponade 3. Cardiomyopathy 4. Cardiogenic shock

ANSWER: 4 The symptoms are indicative of cardiogenic shock (decreased cardiac output leading to inadequate tissue perfusion and initiation of the shock syndrome). Pulmonary embolism, cardiac tamponade, and cardiomyopathy are causes of cardiogenic shock. The cause of this client's cardiogenic shock is a myocardial infarction. ➧ Test-taking Tip: Focus on the issue: life-threatening complications following MI.

85. A client, who is not adhering to a physician's advice about dietary modifications and taking lovastatin (Mevacor®) as prescribed, receives additional teaching from a nurse. Based on the results of the client's serum laboratory results at follow-up, which conclusion by the nurse is accurate? 1. The teaching or the medication is ineffective because the LDL level is elevated. 2. The teaching or the medication is ineffective because the total serum cholesterol is elevated. 3. The teaching and the medication are effective because the HDL level and the triglyceride level are within normal limits. 4. The teaching and the medication are effective because the LDL level is within normal limits and the HDL level is elevated. 33.

ANSWER: 4 The teaching measures and the medication are effective. HMG-CoA reductase inhibitors (statins) reduce LDL cholesterol and increase HDL cholesterol levels. The normal value of LDL cholesterol is less than 130 mg/dL. The goal is to raise HDL cholesterol to 50 mg/dL or higher with the statin antilipidemics. The LDL cholesterol of 100 mg/dL is normal. The total serum cholesterol of 170 mg/dL is normal (normal is less than 200 mg/dL). The HDL is increased above normal, and the statins do not have an effect on triglycerides.

A nurse receives a change-of-shift report for four assigned clients. Which clients should the nurse attend to first? Prioritize the nurse's actions by placing each client in the correct order. _____ A 44-year-old client who has questions about how to empty the Jackson-Pratt drain at home after being discharged tomorrow _____ A 33-year-old client who has a new order to insert a nasogastric (NG) tube and connect to low intermittent suction _____ A usually oriented 76-year-old client diagnosed with thrombophlebitis who has new-onset confusion _____ A 58-year-old client requesting a pain medication for abdominal incision pain rated at a 6 on a 0-10 scale

ANSWER: 4, 3, 1, 2 The client who is confused needs immediate assessment because confusion may be a sign of a complication, such as a stroke or pulmonary embolism. The client in pain should be attended to next because pain can interfere with necessary postoperative activities, such as deep breathing, coughing, and ambulating. The client who has an order for a NG tube insertion should be attended to next. There is no information indicating that this client is nauseated or the purpose of the NG. The last client to be seen is the client who needs teaching. ➧ Test-taking Tip: Use the ABCs (airway, breathing, circulation) to establish the priority client who should be attended to first. Confusion can be indicative of a breathing or circulatory problem. Then follow Maslow's Hierarchy of Needs to address comfort and psychosocial concerns.

A client is experiencing increased dyspnea and chest pain. A physician is notified and orders an electrocardiogram (ECG) to be done stat. The nurse has not done an ECG before and there is no ECG technician in the hospital at this time. Which steps should be taken by the nurse in performing an ECG? Place each answer option into the correct order. ___1__ Obtain the ECG machine ___2__ Place the ECG tracing in the medical record ___3___ Contact an experienced nurse or supervisor forassistance ___4___ Ask an experienced nurse or supervisor to teach the procedure for performing an ECG ___5___ Observe the experienced nurse or supervisor do the ECG ___6___ Call the physician to notify that the ECG is completed ___7___ Review the hospital policy on doing ECGs

ANSWER: 4, 7, 2, 3, 5, 6, 1 The nurse should first review the hospital policy on doing ECGs and ask a nurse with experience for assistance. The new nurse should learn how to do an ECG, as it may be needed in the future, and should ask to learn from the experienced nurse. The nurse should obtain the ECG machine and observe the experienced nurse doing the ECG. The physician should then be notified that the ECG is complete. Finally, the nurse should place the ECG in the medical record. ➧Test-taking Tip: Always follow organizational policies. Know that the nurse is at a legal risk when performing a task that has not been learned. # 16

64. An adult client has laboratory tests drawn during a routine physical examination. A nurse determines that the client has an increased risk for cardiovascular disease. Which laboratory value most likely led the nurse to this conclusion? Laboratory Test Client's Value Normal Parameters White blood 4,900/μL 5000-10,000/μL cell count Red blood 3.8 million/μL 4.2-5.4 million/μL cell count Hemoglobin 11.6 g/dL 12-16 g/dL Hematocrit 36% 37-47% Platelet 149,000/μL 150,000-300,000/μL count Total 210 mg/dL 122-200 mg/dL cholesterol Triglycerides 135 mg/dL 35-135 mg/dL 1. White blood cell count 2. Red blood cell count 3. Hemoglobin and hematocrit 4. Platelet count 5. Total cholesterol 6. Triglyceride level

ANSWER: 5 Only the total cholesterol level is within the risk range. Total cholesterol should be less than 200 mg/dL. Lipid profile studies are part of assessment for cardiovascular disease risk. Elevated triglyceride levels are considered an emerging lipid risk factor. The other abnormal laboratory results are not related to hematological functioning.

An adult client has laboratory tests drawn during a routine physical examination. A nurse determines that the client has an increased risk for cardiovascular disease. Which laboratory value most likely led the nurse to this conclusion? Laboratory Test Client's Value Normal Parameters White blood 4,900/μL 5000-10,000/μL cell count Red blood 3.8 million/μL 4.2-5.4 million/μL cell count Hemoglobin 11.6 g/dL 12-16 g/dL Hematocrit 36% 37-47% Platelet 149,000/μL 150,000-300,000/μL count Total 210 mg/dL 122-200 mg/dL cholesterol Triglycerides 135 mg/dL 35-135 mg/dL 1. White blood cell count 2. Red blood cell count 3. Hemoglobin and hematocrit 4. Platelet count 5. Total cholesterol 6. Triglyceride level

ANSWER: 5 Only the total cholesterol level is within the risk range. Total cholesterol should be less than 200 mg/dL. Lipid profile studies are part of assessment for cardiovascular disease risk. Elevated triglyceride levels are considered an emerging lipid risk factor. The other abnormal laboratory results are not related to hematological functioning. ➧ Test-taking Tip:The key words are "increased cardiovascular risk for cardiovascular disease." Evaluate each test and its relationship to cardiovascular risk (total cholesterol, triglycerides). Determine if the client's laboratory results from these tests is within the normal range.

nurse is caring for a client diagnosed with hypertension. The nurse plans to teach the client progressive muscle relaxation to reduce stress and decrease the client's blood pressure. Which steps should the nurse take when teaching the client to perform progressive muscle relaxation? Prioritize the nurse's actions by placing each step in the correct order. ______ Relax the feet, imagining the tension flowing out with each exhalation. ______ Lie down in a quiet place where you are undisturbed. ______ Contract the muscles of your feet first as you inhale and hold the contraction briefly. ______ Relax your body, allowing it to feel heavy. ______ Lie still for a few minutes after the contraction and relaxation of all muscles. ______ Imagine the tension flowing out with each breath you take. ______ Move up the body, contracting then relaxing each muscle.

ANSWER: 5, 1, 4, 2, 7, 3, 6 The nurse should first instruct the client to lie down in a quiet place where the client will be undisturbed. Then the nurse should instruct the client to relax the body, allowing it to feel heavy. The nurse then tells the client to imagine the tension flowing out with each breath. The client is then instructed to contract the muscles of the feet first as the client inhales and to hold the contraction briefly. The client is then instructed to relax the feet, imagining the tension flowing out with each exhalation. The nurse then tells the client to move up the body, contracting then relaxing each muscle. Finally, the nurse instructs the client to lie still for a few minutes after experiencing the contraction and relaxation of all muscles. Another option is to start with the face first and moving down the body.

Along with persistent, crushing chest pain, which signs/symptoms would make the nurse suspect that the client is experiencing a myocardial infarction? 1. Mid-epigastric pain and pyrosis. 2. Diaphoresis and cool clammy skin. 3. Intermittent claudication and pallor. 4. Jugular vein distention and dependent edema.

Ans: 2

Know what is a Cardio catheterization, how do they do it?

Go through femoral vein etc.-know what to do after the procedure also, don't move or bend leg, check distal pulses

53. A client with a diagnosis of chronic obstructive pulmonary disease (COPD) has developed polycythemia vera, and a nurse has completed teaching on measures to prevent complications. During a home health visit, the nurse evaluates that the client is correctly following the teaching when the client: SELECT ALL THAT APPLY. 1. tells the nurse about discontinuing iron supplements. 2. relays increasing alcohol intake to decrease blood viscosity. 3. records the amount consumed after drinking a glass of water. 4. discusses yesterday's phlebotomy treatment to remove blood. 5. shows the nurse a menu plan for eating three large meals daily. 6. reclines in a recliner chair with legs uncrossed, wearing antiembolic stockings (TEDS®).

Iron supplements, including those in multivitamins, should be avoided because the iron stimulates red blood cell production. Increasing fluid intake to 3,000 mL daily will help decrease blood viscosity. Phlebotomy is performed on a routine or intermittent basis to diminish blood viscosity, deplete iron stores, and decrease the client's ability to manufacture excess erythrocytes. Elevating the legs, avoiding constriction or crossing the legs, and wearing antiembolic socks help prevent deep vein thrombosis. Alcohol increases the risk of bleeding. Frequent, small meals are better tolerated, especially if the liver is involved. ➧ Test-taking Tip: Focus on the issue: measures to prevent complications of polycythemia vera.

nurse is assessing an older adult admitted with a diagnosis of left-sided heart failure and mitral regurgitation. Identify the area with an X where the nurse should place the stethoscope to best auscultate the murmur associated with mitral regurgitation.

The mitral valve is best heard with the bell of the stethoscope at the fifth intercostal space, left midclavicular line. The bell is used to auscultate low-pitched sounds. Abnormalities, such as S3 or S4, are best heard with the bell of the stethoscope. ➧ Test-taking Tip:The words "best" and "mitral" are key words in the stem. A mnemonic for remembering the auscultation points and the location of the heart valves is: "All Points To Monitor." The first letter of each word represents the auscultation point: Aortic valve, Pulmonic valve, Tricuspid valve, and Mitral valve.

16. A nurse is assessing an older adult admitted with a diagnosis of left-sided heart failure and mitral regurgitation. Identify the area with an X where the nurse should place the stethoscope to best auscultate the murmur associated with mitral regurgitation.

The mitral valve is best heard with the bell of the stethoscope at the fifth intercostal space, left midclavicular line. The bell is used to auscultate low-pitched sounds. Abnormalities, such as S3 or S4, are best heard with the bell of the stethoscope. ➧ Test-taking Tip:The words "best" and "mitral" are key words in the stem. A mnemonic for remembering the auscultation points and the location of the heart valves is: "All Points To Monitor." The first letter of each word represents the auscultation point: Aortic valve, Pulmonic valve, Tricuspid valve, and Mitral valve.

Weight is very important-

check daily increase of more than 2lb in a 24 hour period, you have contact Dr. it is not normal. Weigh same time every day with same amt of clothes

CARDIOGENIC SHOCK

chest pain may be present, tachypnea,cyanosis,crackeles, rhonchi,pallor cool clammy skin,anxiety,confusion,agitation tachycarsia,hypotention,narrowed pulse pressure

Difference in ECG for angina you have a

depressed ST segment

hypovolemic shock

inadequate tissue perfusion "all shocks are inadequate tissue perfusion of some sort"

Nursing diagnosis for MI-

ineffective tissue perfusion

CHF-

know paroxysmal nocturnal dyspnea, keep feet flat


Ensembles d'études connexes

Module #6 - Consumer and Producer Surplus

View Set

Management Information Systems: Chapter 3 Information Systems, Organizations, and Strategy

View Set

Seleccionar/ write a question 2.2

View Set

Lifepac History & Geography 1002 Test

View Set

Філософія Рівень1

View Set